testcom1v pd

31
CTO Medicina C/Francisco Silvela, 106 28002 - Madrid Tfno. (0034) 91 782 43 30/33/34 E-mail: [email protected] www. ctomedicina.com 1 Test 1.ª vuelta Distancia Pediatría 1. Señale la afirmación correcta respecto al test de Apgar: 1) Se debe practicar únicamente a los RN patológicos o supues- tamente patológicos. 2) Entre los parámetros que se valoran se encuentra la frecuencia respiratoria. 3) Se debe realizar a los 1-5-10 minutos de vida. 4) Un test de Apgar de 4, al minuto de vida, implica un alto riesgo de parálisis cerebral. 5) La palidez cutánea generalizada supone una puntuación de 1 en el test de Apgar. 2. RN que, al minuto de vida, presenta FC de 110 latidos por minuto, acrocianosis con esfuerzo respiratorio ausente, así como hipotonía y leve mueca al introducir la sonda de aspiración. La puntuación de Apgar será: 1) 1 2) 2 3) 3 4) 4 5) 5 3. ¿Cuál de las siguientes afirmaciones sobre los cuidados del recién nacido prematuro es FALSA? 1) La alimentación debe iniciarse siempre por sonda nasogástrica en los menores de 32 semanas de edad gestacional, por la falta de coordinación entre succión, deglución y respiración. 2) En los casos que necesiten tratamiento con eritropoyetina, debe iniciarse a la vez tratamiento con hierro vía oral. 3) Las necesidades de líquidos son mayores que en los neonatos a término, tanto más cuanto más prematuro sea el recién nacido. 4) Debe aumentarse el intervalo de dosis de todos los medi- camentos de eliminación renal. 5) Hay que realizar a todos ecografía transfontanelar aunque no tengan alteraciones en la exploración neurológica. 4. Señale la afirmación FALSA respecto a las lesiones producidas por traumatismos obstétricos: 1) El cefalohematoma es una hemorragia subperióstica, y está delimitado por las suturas craneales. 2) La clavícula es el hueso que se fractura con mayor frecuencia durante el parto. 3) El llamado “nódulo de Stroemayer” suele requerir trata- miento quirúrgico. 4) La rotura hepática no siempre precisa tratamiento quirúrgico. 5) Las fracturas craneales más frecuentes en el parto son las lineales. 5. ¿Cuál de los siguientes fenómenos dermatológicos que podemos encontrar en un neonato NO debe sugerirnos una patología subyacente? 1) Manchas hipopigmentadas geográficas. 2) Ictericia precoz en las primeras 24 horas. 3) Nevus flammeus. 4) Melanosis pustulosa. 5) Manchas café con leche. 6. Ante un neonato a término que tras cesárea presenta precoz- mente taquipnea y cianosis que desaparece con pequeñas cantidades de oxígeno, en el que la auscultación pulmonar es normal y en el que la radiografía muestra marcas vasculares prominentes y líquido en las cisuras sin broncograma aéreo, sospecharemos: 1) Enfermedad de membrana hialina (EMH). 2) Taquipnea transitoria del recién nacido. 3) Síndrome de aspiración meconial (SAM). 4) Persistencia de la circulación fetal (PCF). 5) Displasia broncopulmonar (DBP). 7. RN de 35 semanas de edad gestacional que en las primeras 12 horas de vida presenta taquipnea, quejido audible y cianosis progresiva, refractaria a la administración de oxígeno. A la auscultación encontramos crepitantes, sobre todo en bases pulmonares. En la gasometría aparece hipoxemia progresiva, aumento de la PCO 2 y acidosis metabólica. ¿Qué radiografía de tórax esperaría encontrar en este momento? 1) Normal. 2) Infiltrado de patrón reticulogranular con broncograma aéreo. 3) Hiperinsuflación y aumento de las marcas vasculares, sin broncograma. 4) Patrón de esponja.

Upload: mariana-villegas-p

Post on 05-Aug-2015

78 views

Category:

Documents


10 download

TRANSCRIPT

Page 1: Testcom1v Pd

Test 1.ª vuelta Distancia

CTO Medicina • C/Francisco Silvela, 106 • 28002 - Madrid • Tfno. (0034) 91 782 43 30/33/34 • E-mail: [email protected] • www. ctomedicina.com 1

Pediatría

Test 1.ª vuelta Distancia

Pediatría

1. Señale la afi rmación correcta respecto al test de Apgar:

1) Se debe practicar únicamente a los RN patológicos o supues-tamente patológicos.

2) Entre los parámetros que se valoran se encuentra la frecuencia respiratoria.

3) Se debe realizar a los 1-5-10 minutos de vida.4) Un test de Apgar de 4, al minuto de vida, implica un alto

riesgo de parálisis cerebral.5) La palidez cutánea generalizada supone una puntuación

de 1 en el test de Apgar.

2. RN que, al minuto de vida, presenta FC de 110 latidos por minuto, acrocianosis con esfuerzo respiratorio ausente, así como hipotonía y leve mueca al introducir la sonda de aspiración. La puntuación de Apgar será:

1) 12) 23) 34) 45) 5

3. ¿Cuál de las siguientes afi rmaciones sobre los cuidados del recién nacido prematuro es FALSA?

1) La alimentación debe iniciarse siempre por sonda nasogástrica en los menores de 32 semanas de edad gestacional, por la falta de coordinación entre succión, deglución y respiración.

2) En los casos que necesiten tratamiento con eritropoyetina, debe iniciarse a la vez tratamiento con hierro vía oral.

3) Las necesidades de líquidos son mayores que en los neonatos a término, tanto más cuanto más prematuro sea el recién nacido.

4) Debe aumentarse el intervalo de dosis de todos los medi-camentos de eliminación renal.

5) Hay que realizar a todos ecografía transfontanelar aunque no tengan alteraciones en la exploración neurológica.

4. Señale la afi rmación FALSA respecto a las lesiones producidas por traumatismos obstétricos:

1) El cefalohematoma es una hemorragia subperióstica, y está delimitado por las suturas craneales.

2) La clavícula es el hueso que se fractura con mayor frecuencia durante el parto.

3) El llamado “nódulo de Stroemayer” suele requerir trata-miento quirúrgico.

4) La rotura hepática no siempre precisa tratamiento quirúrgico.5) Las fracturas craneales más frecuentes en el parto son las

lineales.

5. ¿Cuál de los siguientes fenómenos dermatológicos que podemos encontrar en un neonato NO debe sugerirnos una patología subyacente?

1) Manchas hipopigmentadas geográfi cas.2) Ictericia precoz en las primeras 24 horas.3) Nevus fl ammeus.4) Melanosis pustulosa.5) Manchas café con leche.

6. Ante un neonato a término que tras cesárea presenta precoz-mente taquipnea y cianosis que desaparece con pequeñas cantidades de oxígeno, en el que la auscultación pulmonar es normal y en el que la radiografía muestra marcas vasculares prominentes y líquido en las cisuras sin broncograma aéreo, sospecharemos:

1) Enfermedad de membrana hialina (EMH).2) Taquipnea transitoria del recién nacido.3) Síndrome de aspiración meconial (SAM).4) Persistencia de la circulación fetal (PCF).5) Displasia broncopulmonar (DBP).

7. RN de 35 semanas de edad gestacional que en las primeras 12 horas de vida presenta taquipnea, quejido audible y cianosis progresiva, refractaria a la administración de oxígeno. A la auscultación encontramos crepitantes, sobre todo en bases pulmonares. En la gasometría aparece hipoxemia progresiva, aumento de la PCO

2 y acidosis metabólica. ¿Qué radiografía de

tórax esperaría encontrar en este momento?

1) Normal.2) Infi ltrado de patrón reticulogranular con broncograma aéreo.3) Hiperinsufl ación y aumento de las marcas vasculares, sin

broncograma.4) Patrón de esponja.

Page 2: Testcom1v Pd

Test 1.ª vuelta Distancia

CTO Medicina • C/Francisco Silvela, 106 • 28002 - Madrid • Tfno. (0034) 91 782 43 30/33/34 • E-mail: [email protected] • www. ctomedicina.com 2

Pediatría5) Borde de ambos pulmones colapsados.

8. En el tratamiento de la EMH se incluyen las siguientes medidas, EXCEPTO:

1) Administración de surfactante endotraqueal.2) Antibioterapia.3) Administración de oxígeno y ventilación mecánica, si es

preciso.4) Prostaglandinas.5) Control del aporte de líquidos.

9. Un niño de 42 semanas de gestación, arrugado, pálido, hipotó-nico, apneico y cubierto de líquido amniótico verdoso. Respecto al cuadro que puede padecer este paciente, señale la opción FALSA:

1) La primera maniobra en su reanimación debe ser la aspiración de tráquea bajo visión laringoscópica.

2) Es un patología típica de RNPT y RNT.3) E. coli es el germen más frecuente de infección bacteriana

en estos pacientes.4) En la Rx de tórax es típico encontrar un patrón de atrapa-

miento aéreo.5) Esta patología produce una llamativa hipertensión pulmonar.

10. Recién nacido de 38 semanas que tras expulsivo difi cultoso presenta el brazo izquierdo pegado al tronco junto con cierta rotación interna de dicho miembro. Señale la respuesta verda-dera respecto a la lesión obstétrica que sospecha en este recién nacido:

1) El refl ejo de Moro no estará presente en el lado izquierdo.2) Se debe a una lesión en las raíces inferiores del plexo braquial.3) Este tipo de parálisis es menos frecuente que la parálisis de

Klumpke.4) Es característica la presencia de mano caída.5) Suele asociarse con síndrome de Horner.

11. Un RN de 32 semanas, con antecedente de enfermedad de membrana hialina sometido a ventilación mecánica con altas concentraciones de oxígeno, tiene actualmente un mes de vida y sigue dependiendo del respirador para mantener adecuadas saturaciones de oxígeno. En los últimos días presenta además oliguria con edemas en miembros inferiores y se palpa el hígado a 5 cm bajo el reborde costal derecho. En la Rx de tórax se observa una imagen de pequeñas zonas redondas y claras que alternan con otras de mayor densidad. En relación con la patología que presenta este paciente, señale la respuesta FALSA:

1) Se consideran factores de riesgo para su desarrollo tener distrés respiratorio severo que precise largos períodos de administración de O

2.

2) Las principales causas de muerte de estos pacientes son la bronquiolitis necrotizante y fallo cardíaco derecho.

3) El tratamiento incluye broncodilatadores, diuréticos y dexa-metasona.

4) La nefrolitiasis puede ser una complicación de estos pacientes, secundaria al uso de diuréticos y a la alimentación parenteral.

5) A pesar de que pueda retirarse la administración de O2 antes

de salir de la unidad de cuidados intensivos, el pronóstico para la mayoría de estos pacientes es malo, presentando en la adolescencia obstrucción de las vías respiratorias e hiperreactividad bronquial.

12. Recién nacido a término que presenta difi cultad respiratoria inmediata, con importante tiraje intercostal y supraesternal, taquipnea, tórax hiperinsufl ado y abdomen excavado. En la auscultación pulmonar destaca abolición casi total del mur-mullo vesicular en el hemitórax izquierdo, con tonos cardíacos desplazados a la derecha. Señale la respuesta verdadera acerca de la patología que sospecha:

1) En más del 50% de los casos hay otras malformaciones asociadas.

2) El tratamiento es quirúrgico, y debe realizarse lo antes posible, tratando después las alteraciones que haya asociadas.

3) El diagnóstico diferencial con malformaciones pulmonares quísticas se hace con ecografía.

4) El refl ujo gastroesofágico es una secuela rara tras la cirugía.5) Los afectados suelen desarrollar hipertensión pulmonar a

partir del nacimiento, que en los casos graves condiciona su pronóstico.

13. RNPT de 29 semanas de edad gestacional sufre distrés respi-ratorio desde el nacimiento. A las 36 horas de vida presenta hipotensión, bradicardia, cianosis y la fontanela se encuentra abombada. En el hemograma presenta 15.000 leucocitos, neu-trófi los 45%, linfocitos 42%, cayados 0%, plaquetas 180.000, hematocrito 30%. ¿Qué prueba complementaria consideraría más adecuada en este momento?

1) Punción lumbar y análisis del LCR.2) Ecocardiograma.3) Ecografía cerebral.4) TC craneal.5) Rx tórax.

14. Un RN pretérmino de 1.800 g de peso, con antecedentes de hipoxia perinatal, a los 10 días de vida presenta distensión abdominal importante, vómitos y deposiciones hemorrágicas. En la Rx de abdomen se observa edema de asas intestinales, con un patrón en “miga de pan” y presencia de gas en la pared intestinal. ¿Cuál sería la opción terapéutica más adecuada en este paciente?

1) Enema de solución hiperosmolar.2) Alimentación enteral a débito continuo a través de una

sonda nasogástrica.3) Actitud expectante y vigilancia cuidadosa, ante la posibilidad

de perforación intestinal.4) Se debe suspender la alimentación enteral y pautar fl uido-

terapia y antibioterapia i.v.5) La presencia de neumatosis intestinal aconseja la interven-

ción quirúrgica urgente.

15. RN de 38 semanas de edad gestacional presenta distensión abdominal y vómitos biliosos durante el primer día de vida. En el examen físico se palpan cordones duros que siguen el marco cólico. Una radiografía en bipedestación muestra una masa en

Page 3: Testcom1v Pd

Test 1.ª vuelta Distancia

Pediatría

CTO Medicina • C/Francisco Silvela, 106 • 28002 - Madrid • Tfno. (0034) 91 782 43 30/33/34 • E-mail: [email protected] • www. ctomedicina.com 3

“pompa de jabón” en cuadrante inferior derecho. El tratamiento inmediato más apropiado es:

1) Antibioterapia.2) Supositorios de glicerina.3) Enema de solución hiperosmolar.4) Estimulación rectal.5) Cirugía.

16. Señale la afi rmación FALSA, de entre las siguientes, respecto a la ictericia fi siológica:

1) Las cifras máximas de bilirrubina suelen alcanzarse a los 3-4 días.

2) La ictericia en los RNPT suele ser de inicio algo más tardío y más prolongada que en los RNT.

3) Puede comenzar en las primeras 24 horas de vida en el RNT.4) Su duración suele ser inferior a 10-15 días.5) En los RNPT pueden alcanzarse cifras de hasta 14 mg/dl.

17. En relación a la ictericia por lactancia materna o síndrome de Arias es FALSO que:

1) Su frecuencia es aproximadamente de 1 por cada 200 RN que reciben lactancia materna.

2) Es una ictericia que aparece a la mitad o fi nal de la primera semana, alcanzando su máximo en la tercera.

3) Se pueden alcanzar cifras de hasta 30 mg/dl, de ahí el riesgo de kernicterus en estos neonatos.

4) En la patogenia se implican varias sustancias presentes en la leche materna como pregnanodiol, ácidos grasos de cadena larga y glucuronidasas.

5) Cuando se suspende la lactancia materna durante 2-4 días se produce una rápida disminución de la bilirrubina sérica.

18. En relación a la enfermedad hemolítica por isosensibilización AB0, señale la FALSA:

1) La madre suele ser 0.2) Los anticuerpos causantes suelen ser de la clase IgG.3) Pueden resultar afectos los productos del primer embarazo.4) El antígeno más frecuentemente implicado es A1.5) La enfermedad se agrava en los embarazos siguientes, si no

hacemos profi laxis.

19. Un lactante de una semana es llevado a Urgencias en estado de coma con hemorragias en la retina y palidez importante. Entre sus antecedentes destaca que el parto tuvo lugar en el domi-cilio, no presentó complicaciones posteriores y está recibiendo lactancia materna. Señale lo FALSO respecto a este cuadro:

1) Se debe a un défi cit de los factores dependientes de vitamina K.

2) Suele iniciarse a partir de las primeras 24 horas.3) Es más frecuente en hijos de madres que han tomado feni-

toína o fenobarbital.4) Las complicaciones hemorrágicas son menos frecuentes en

niños que toman LM que en los que reciben biberón.5) Se trata con la administración de vitamina K y/o plasma

fresco congelado.

20. Respecto a la anemia en el período neonatal, señale cuál de las siguientes afi rmaciones es FALSA:

1) La anemia fi siológica es más intensa y más precoz en los RNT que en los RNPT.

2) El cuadro de anemia fi siológica neonatal está en relación con la hemólisis de glóbulos rojos y con un défi cit de eritropoyetina.

3) En los RNPT está indicada la administración de suplementos de hierro a partir de los 2 meses de vida.

4) La anemia fi siológica puede precisar transfusión en algunos casos.

5) La betatalasemia no se manifestará antes de los 6 meses de vida.

21. Señale la afi rmación FALSA, de entre las siguientes, respecto a la isoinmunización Rh:

1) La enfermedad hemolítica grave suele afectar al segundo hijo.2) Suele cursar con ictericia, que aparece en las primeras 24

horas de vida.3) Debe hacerse profi laxis con gammaglobulina específi ca

anti-D antes de que pasen 72 horas del parto, cuando el test de Coombs directo es positivo.

4) Cuando las cifras de bilirrubina superan los 20 mg/dl, es necesario realizar una exanguinotransfusión.

5) El 90% de los casos de incompatibilidad se deben al antí-geno D.

22. Respecto a la hipoglucemia del hijo de madre diabética, señale cuál de las siguientes afi rmaciones es FALSA:

1) Se observa en el 75% de los hijos de madres diabéticas y en el 25% de los hijos de madres con diabetes gestacional.

2) Aparece en las primeras 24 horas de vida y suele ser asinto-mática.

3) Se piensa que es debida a hiperinsulinismo fetal.4) La probabilidad de que el RN presente hipoglucemia es más

elevada cuanto más altas se encuentren las cifras de glucosa en sangre de cordón.

5) Se trata mediante infusión i.v. de bolos de glucosa y debe retrasarse la alimentación oral para minimizar el riesgo de hipoglucemia de rebote.

23. NO es propio del RN hijo de madre diabética:

1) Megalencefalia.2) Síndrome asfíctico.3) Hiperbilirrubinemia.4) Síndrome de regresión caudal.5) Cardiomiopatía.

24. En la sepsis del RN, los gérmenes más frecuentes son:

1) Klebsiella.2) E. coli y estreptococos del grupo A.3) E. coli y estreptococos del grupo B.4) Estafi lococo.5) Gardnerella vaginalis.

25. ¿Qué haría usted ante un RN de madre con hepatitis B activa en el momento del nacimiento?

Page 4: Testcom1v Pd

Test 1.ª vuelta Distancia

CTO Medicina • C/Francisco Silvela, 106 • 28002 - Madrid • Tfno. (0034) 91 782 43 30/33/34 • E-mail: [email protected] • www. ctomedicina.com 4

Pediatría30. Un RN hijo de madre prostituta presenta en el 7.º día de vida un

cuadro de fi ebre con letargia y rechazo de las tomas así como convulsiones. A la exploración destaca hepatoesplenomegalia con fontanela anterior abombada y lesiones vesiculares en ca-lota craneal. ¿Cuál sería la causa más probable de este cuadro?

1) Infección por VIH.2) Infección por CMV.3) Infección por VHS tipo I.4) Infección por VHS tipo II.5) Exantema tóxico del RN.

31. Un recién nacido de 3 semanas de vida que desprendió el cordón umbilical a los 16 días y cuyo ombligo no ha cicatrizado, pre-senta una pequeña tumoración sonrosada con secreción sero-mucosa con un pH de 7,4. ¿Cuál de los siguientes diagnósticos le parece más probable?

1) Onfalocele.2) Granuloma umbilical.3) Quiste o persistencia del uraco.4) Gastrosquisis.5) Ombligo amniótico.

32. Recién nacida, de 3 semanas de vida, que presenta valores de TSH de 75 μg/ml (0,3 –5 ) en el cribado neonatal de hipotiroidismo. Con respecto a esta situación, es FALSO:

1) Su causa más frecuente es la disgenesia tiroidea.2) Situaciones de dismorfogénesis pueden asociarse a sordera

neurosensorial.3) Un tratamiento precoz supone un mejor pronóstico neuro-

lógico.4) Los signos clínicos del hipotiroidismo están ya presentes

inmediatamente al nacimiento.5) En situaciones de mal control pueden acabar desencade-

nando una pubertad precoz.

33. En relación con el desarrollo psicomotor en el niño, señale la respuesta FALSA:

1) La sonrisa social referencial aparece ente el 1.º y 2.º mes.2) El sostén cefálico se inicia a los 3 meses.3) La sedestación estable se consigue a los 6-8 meses.4) Emite sus primeros fonemas con 3-4 semanas.5) Realiza frases cortas con 2 años.

34. En relación con el crecimiento y el desarrollo del niño en su primer año de vida, ¿cuál de las siguientes afi rmaciones es FALSA?

1) El niño aproximadamente duplica su peso al 5.º mes y lo triplica al año de vida.

2) El peso del RN puede disminuir un 5-10% en la primera semana de vida por pérdida de líquido extravascular.

3) Los refl ejos precoces o arcaicos presentes al nacimiento suelen desaparecer entre los 3 y 4 meses de edad.

4) Hacia los 2 meses suele haber hecho su aparición la sonrisa social.

5) Duplican la talla del nacimiento poco después de cumplir el primer año de edad.

1) Observar la aparición de los síntomas y luego tratar.2) Extraer sangre de cordón para estudiar transaminasas.3) Exanguinotransfusión inmediata, si la madre tiene el antí-

geno HBe positivo.4) Poner una dosis bimensual de gammaglobulina.5) Iniciar vacunación y gammaglobulina en paritorio.

26. Un RN de bajo peso para su edad gestacional presenta un exantema petequial, hepatoesplenomegalia y coriorretinitis. En la TC craneal se observan calcifi caciones periventriculares sin otros hallazgos. Con respecto al cuadro que parece presentar este paciente, señale la verdadera:

1) Es la segunda causa más frecuente de infección viral con-génita.

2) El 90% de los recién nacidos infectados estarán asintomá-ticos al nacer.

3) La manifestación más tardía y característica es una hipoacusia de transmisión.

4) La afectación ocular típica es una retinitis pigmentosa.5) Estos niños suelen desarrollar macrocefalia evolutivamente.

27. ¿Cuál de las siguientes manifestaciones NO forma parte de la tétrada de Sabin?

1) Coriorretinitis.2) Cataratas.3) Crisis convulsivas.4) Hidrocefalia.5) Calcifi caciones cerebrales.

28. Un RN de 38 semanas de edad gestacional y 1.750 g, presenta erupción purpuropetequial, hepatoesplenomegalia y se ausculta un soplo continuo áspero en 2.º espacio intercostal izquierdo. En la Rx de tórax se objetiva cardiomegalia con aumento de la vascularización pulmonar así como imágenes lineales radiotransparentes con aumento de densidad ósea en la zona metafi saria del húmero sin reacción perióstica. En relación con el cuadro que presenta este paciente, señale la respuesta FALSA:

1) Cuando la madre contrae la infección en el tercer trimestre aumenta el riesgo de infección del feto, pero el cuadro clínico es más leve.

2) Si la infección se produce después de la 12.ª semana de gestación, es poco probable la asociación de una cardiopatía congénita.

3) La presencia de IgM específi ca antirrubéola refl eja la pro-ducción en el útero de anticuerpos por el feto.

4) El lactante puede permanecer con infección crónica durante meses después del nacimiento.

5) El virus puede aislarse en sangre.

29. De los siguientes, el factor más relacionado con el crecimiento intrauterino retardado tipo simétrico es la:

1) Infección congénita.2) Isoinmunización Rh.3) Hipertensión.4) Diabetes con vasculopatía.5) Edad de la mujer por encima de 35 años.

Page 5: Testcom1v Pd

Test 1.ª vuelta Distancia

Pediatría

CTO Medicina • C/Francisco Silvela, 106 • 28002 - Madrid • Tfno. (0034) 91 782 43 30/33/34 • E-mail: [email protected] • www. ctomedicina.com 5

4) Laringitis estridulosa.5) Epiglotitis.

40. Ante un niño de 2 años de edad que, estando previamente bien, se despierta bruscamente por la noche con tos perruna, estridor y difi cultad respiratoria, pulso acelerado y afebril, pensaremos en:

1) Laringotraqueítis aguda.2) Laringitis espasmódica.3) Epiglotitis aguda.4) Difteria.5) Traqueítis bacteriana.

41. Varón de 5 años que, estando previamente bien, despierta por la noche con fi ebre de 39,8 º C, voz apagada, intenso babeo, estridor inspiratorio con escasa tos. A la exploración destaca mal estado general, estridor inspiratorio e intenso tiraje supra-esternal y subcostal, así como tendencia a la hiperextensión del cuello. Se realiza radiografía lateral de faringe, observando ocupación de espacio supraglótico. En el hemograma destaca una leucocitosis con desviación izquierda. ¿Qué medida terapéutica NO estaría aconsejada en este caso?

1) Aplicar oxígeno e intentar obtener una vía aérea artifi cial en quirófano.

2) Iniciar tratamiento antibiótico con ceftriaxona.3) Aplicar adrenalina racémica y corticoides.4) Usar las medidas generales para descender la hipertermia

que sufre el paciente.5) Mantener tranquilo al paciente.

42. Lactante varón de 3 meses que desde hace dos días presenta cuadro de rinorrea serosa con estornudos y tos seca acompañado de rechazo de las tomas. Hoy acude a Urgencias por cuadro de difi cultad respiratoria con taquipnea y tos sibilante paroxística. Exploración física: 37,7 ºC, regular estado general, acianótico, con aleteo nasal y tiraje subcostal e intercostal. Auscultación pul-monar: sibilancias espiratorias difusas con espiración alargada. Se realiza hemograma con 3,5 millones de hematíes; Hb de 10 g/dl; Hto de 32%; 7.000 leucocitos/mm3 con 50% neutrófi los, 35% linfocitos y 5% monocitos. En la Rx de tórax destaca hipe-rinsufl ación pulmonar. Señale la FALSA respecto a la evolución clínica de esta enfermedad:

1) La fase más crítica son las primeras 48-72 horas desde el comienzo de la tos y la disnea.

2) En menores de un mes puede cursar como apnea con escasez de clínica respiratoria.

3) Después de la fase crítica mejoran muy rápidamente.4) La ribavirina podría usarse en lactantes con cardiopatías

congénitas o displasia broncopulmonar.5) Las complicaciones bacterianas como la bronconeumonía

u OMA son frecuentes.

43. Paciente de 2 años de edad consulta por estancamiento ponde-roestatural (< P3). Entre los antecedentes personales destacan en el último año episodios repetidos de bronquiolitis con regular recuperación posterior, persistiendo una tos crónica. Además presenta diarrea intermitente sin acompañarse de fi ebre ni vómitos. ¿Cuál sería el diagnóstico más probable?

35. Con respecto al calostro, es FALSO que:

1) Se denomina así a la secreción mamaria durante los primeros 2-4 días postparto.

2) Contiene más proteínas y más carbohidratos que la leche humana madura.

3) Su densidad es mayor que la de la leche humana madura.4) Las características de leche humana madura se adquieren

alrededor de la 3.ª-4.ª semana.5) Posee factores inmunológicos.

36. Respecto a las diferencias entre la leche materna y la leche de vaca, señale la afi rmación correcta:

1) La leche de vaca tiene un contenido energético dos veces superior al de la leche materna.

2) La leche de vaca contiene una cantidad de calcio superior a la leche materna.

3) La leche materna contiene mayor cantidad de vitamina K que la leche de vaca.

4) La leche de vaca es más rica en hidratos de carbono que la leche materna.

5) La leche materna tiene una relación caseína/seroproteínas de 70/30.

37. La estatura de un niño de 4 años está por debajo del percentil 3. Su velocidad de crecimiento es normal. La edad ósea es de 2 años. El padre presentó un retraso en la pubertad, aunque actualmente su talla está dentro de los límites normales. Este proceso se denomina:

1) Hipopituitarismo primario.2) Hipopituitarismo secundario.3) Retraso constitucional del crecimiento.4) Talla baja genética.5) Síndrome de Laron.

38. Acude a su consulta un padre con su hijo de 8 años preocupado porque dice que su hijo es el niño más bajo de toda su clase. Su talla actual se encuentra en el percentil 2 y su talla genética está situada en el percentil 3. Al realizar el estudio de la edad ósea del niño se ve que ésta coincide con su edad cronológica. ¿Qué cree usted que explicaría la talla baja de este niño?

1) Un hipocrecimiento por defi ciencia de la hormona de cre-cimiento.

2) Una alteración cromosómica.3) Un hipotiroidismo congénito.4) Un retraso constitucional del crecimiento.5) Una talla baja familiar.

39. Niña de 2 años que acude a Urgencias por cuadro de disnea, estridor y tos perruna. En la exploración destaca rinorrea intensa, amígdalas hipertrófi cas con exudado puntiforme. Fiebre de 38,5º C. A la auscultación pulmonar se aprecia hipo-ventilación bilateral y estridor inspiratorio. El diagnóstico más probable será:

1) Traqueítis bacteriana.2) Laringitis supraglótica.3) Laringitis aguda.

Page 6: Testcom1v Pd

Test 1.ª vuelta Distancia

CTO Medicina • C/Francisco Silvela, 106 • 28002 - Madrid • Tfno. (0034) 91 782 43 30/33/34 • E-mail: [email protected] • www. ctomedicina.com 6

Pediatría1) No suele haber antecedentes familiares de atopia.2) La proteína comúnmente implicada es la betalactoglobulina.3) Las pruebas cutáneas serán positivas.4) El diagnóstico defi nitivo se realiza mediante un test de

supresión-provocación.5) Suele ser un proceso transitorio.

49. RN de pocas horas de vida que presenta exceso de secreciones orales, episodios de cianosis y atragantamiento durante las tomas junto con un abdomen muy distendido y timpánico a la percusión. Entre sus antecedentes destacan polihidramnios materno durante la gestación e imposibilidad de pasarle una sonda nasogástrica hasta el estómago. En relación con la pato-logía que sospecha, refi era cuál de las complicaciones siguientes al tratamiento quirúrgico es la más frecuente:

1) Fístula anastomótica.2) Estenosis esofágica.3) Fístula traqueoesofágica recidivante.4) Refl ujo gastroesofágico.5) Traqueomalacia.

50. Varón de 3 meses que consulta por vómitos desde hace un mes. Vomita después de todas las tomas, sin fuerza; come siempre con buen apetito y presenta adecuado desarrollo ponderoes-tatural, sin pérdida de peso. Realiza dos deposiciones diarias con heces de consistencia, aspecto y volumen normal. Entre sus antecedentes destaca estar alimentado con fórmula artifi cial desde el mes de vida. Su actitud sería:

1) Cambiar su fórmula por un hidrolizado de proteínas de vaca.2) Hacer endoscopia para descartar esofagitis.3) Solicitar ecografía abdominal y/o tránsito digestivo.4) Realizar manometría esofágica.5) Indicar a la madre medidas posturales y usar espesante de

la leche.

51. Señale la respuesta verdadera respecto a la estenosis hipertrófi ca de píloro:

1) Suele ocurrir en la primera semana de vida.2) Es más frecuente en niñas.3) Presenta alcalosis metabólica hiperclorémica con hipopo-

tasemia.4) La prueba diagnóstica de elección es la Rx de abdomen.5) El tratamiento consiste en la pilorotomía submucosa de

Ramstedt.

52. RN con síndrome de Down presenta vómitos biliosos a las 36 horas de vida. A la exploración destaca abdomen excavado. El diagnóstico más probable es:

1) Estenosis hipertrófi ca de píloro.2) Enfermedad de Hirschprung.3) Divertículo de Meckel.4) Atresia duodenal.5) Invaginación intestinal.

53. Con respecto al divertículo de Meckel, es FALSO que:

1) Afecta al 1-2% de la población.

1) Enfermedad celíaca.2) Tuberculosis.3) Asma.4) Fibrosis quística.5) Défi cit de alfa-1 antitripsina.

44. Sobre la traqueítis bacteriana, todo es cierto EXCEPTO:

1) La causa más frecuente es Haemophilus infl uenzae.2) Debe sospecharse ante todo crup viral con fi ebre que no

responde al tratamiento.3) No estarán presentes los hallazgos clínicos de una epiglotitis.4) Será necesario instaurar tratamiento antibiótico.5) En ocasiones puede requerir uso de vía aérea artifi cial.

45. Señale la respuesta FALSA, al hablar de la fi brosis quística:

1) La lesión anatomopatológica pulmonar más precoz es la bronquiolitis.

2) Un 10% de los casos debuta con un íleo meconial.3) La función endocrina del páncreas se altera con el tiempo.4) La aspergilosis broncopulmonar alérgica es excepcional en

estos pacientes y se trata con antifúngicos como el vorico-nazol.

5) Pueden presentar deshidratación con alcalosis hipoclorémica en ambientes calurosos.

46. En el tratamiento de la fi brosis quística, NO es cierto que:

1) La fi sioterapia respiratoria alarga la supervivencia.2) Los productos enzimáticos micronizados disminuyen la

esteatorrea.3) Se necesita aportar suplementos de vitaminas liposolubles

a la dieta.4) La colonización de la vía respiratoria por P. aeruginosa es

fácilmente revertida con la antibioterapia adecuada.5) En ocasiones son necesarios lavados y aspiración traqueo-

bronquial broncoscópica.

47. Varón de 13 meses que desde hace 2 días presenta fi ebre de 38 ºC y síntomas catarrales. Hoy acude por presentar vómitos no biliosos y 6 deposiciones en las últimas 2 horas, líquidas, no fétidas, sin moco ni pus ni sangre. Respecto al cuadro que pre-senta este paciente, señale cuál de las siguientes afi rmaciones es FALSA:

1) El agente causal más frecuente en la infancia es el rotavirus.2) Suele ser un proceso autolimitado, que cede en 3-10 días.3) Puede causar deshidratación isotónica con acidosis.4) Es frecuente que tienda a la cronifi cación.5) El diagnóstico puede hacerse mediante detección rápida

de antígeno de rotavirus en heces.

48. Lactante de 3 meses, alimentado al pecho hasta hace una semana en que, debido a una hipogalactia, se decide suplementar con una fórmula artifi cial. Hoy acude a Urgencias por presentar anorexia, vómitos, pérdida de peso así como exantema urti-carial en tronco. Entre sus antecedentes destaca: RNT, nacido por cesárea y haber iniciado alimentación oral a las 4 horas de vida con fórmula artifi cial. Con respecto a la patología actual, es FALSO que:

Page 7: Testcom1v Pd

Test 1.ª vuelta Distancia

Pediatría

CTO Medicina • C/Francisco Silvela, 106 • 28002 - Madrid • Tfno. (0034) 91 782 43 30/33/34 • E-mail: [email protected] • www. ctomedicina.com 7

57. Ante una enferma con clínica sospechosa de enfermedad celíaca, señale lo correcto respecto a su diagnóstico:

1) Ante la sospecha muy alta con clínica fl orida, se debe excluir el gluten de la dieta, sin necesidad de realizar biopsia.

2) Los Acs antitransglutaminasa son los más específi cos.3) La anatomía patológica de la muestra de biopsia es patog-

nomónica.4) Antes de realizar la primera biopsia, es necesario excluir el

gluten.5) El control de la enfermedad se basa en la realización de

biopsias anuales.

58. Neonato varón, de 3 semanas de vida, acude a su consulta por ictericia con coluria e hipocolia. A la exploración se palpa hepa-tomegalia. En la gammagrafi a con HIDA, la captación es normal, pero la excreción es nula. Sobre la entidad que sospecha, señale la opción FALSA:

1) Se puede asociar a poliesplenia.2) El tratamiento defi nitivo es la hepatoportoenterostomía.3) En la biopsia es posible hallar células gigantes.4) Es infrecuente la aparición de varios casos en una misma

familia.5) En el sondaje duodenal se suele observar ausencia de bilis.

59. Paciente de 5 meses que acude por presentar, desde hace 2 días, hasta 10 deposiciones al día, líquidas sin moco ni sangre, así como dolor medio abdominal sin relación temporal. Entre sus antecedentes destaca diarrea sanguinolenta hace una semana. En la exploración destaca: afebril, buen estado general, disten-sión abdominal con abundantes ruidos hidroaéreos y eritema en región del pañal. En el examen macroscópico de heces se detecta presencia de azúcares reductores. Este cuadro es:

1) Défi cit de sacarasa-isomaltasa.2) Défi cit aislado de isomaltasa.3) Défi cit de lactasa.4) Défi cit de enteroquinasa.5) Gastroenteritis por rotavirus.

60. Lactante de un mes de edad, nacido tras un embarazo controlado y normal, a las 35 semanas de edad gestacional, que acude a su consulta para la “revisión del niño sano” correspondiente. Todo parece estar bien salvo que los testículos se encuentran en el canal inguinal, y aunque sí lo hacen con tracción manual, espontámente no bajan al escroto. ¿Cuál es la actitud más ade-cuada en este momento?

1) Si la alteración es bilateral, hay que consultar con el cirujano

cuanto antes, para que realice una orquidopexia, por el riesgo de esterilidad y degeneración maligna.

2) Hay que esperar, porque es posible que espontánemente se complete el descenso testicular.

3) Trataría con HCG vía intramuscular a días alternos, y si no hay respuesta, mandaría al cirujano para la extirpación de ambos testes.

4) Se trata de unos testículos en ascensor, lo que no requiere ningún tipo de tratamiento ni seguimiento.

5) Trataría con testosterona y HCG, y si no hay respuesta, con-sultaría con los cirujanos para su descenso.

2) Se localiza en íleon a unos 75 cm de la válvula ileocecal.3) Se denomina hernia de Littré cuando el divertículo de Meckel

se aloja en una hernia inguinal indirecta.4) Su manifestación clínica más frecuente es una hemorragia

gastrointestinal acompañada de dolor abdominal.5) El método diagnóstico más útil es la gammagrafía con

Tc-99.

54. Un varón de 14 meses, previamente sano, de forma brusca presenta crisis de llanto con encogimiento de piernas y palidez, separados por períodos asintomáticos de duración variable en los que permanece decaído. Ha presentado dos vómitos y una deposición semilíquida sanguinolenta. Exploración física: afebril; palpación de una masa alargada y mal defi nida, localizada en hipocondrio derecho. En la Rx de abdomen aparece silencio aéreo en hipocondrio derecho. Señale cuál de las siguientes afi rmaciones es cierta respecto al cuadro que presenta este paciente:

1) Debe retrasarse el tratamiento durante 4-6 horas debido a la alta incidencia de reducción espontánea.

2) La recidiva es menos frecuente después de la reducción hidrostática que tras la reducción quirúrgica.

3) Si hay signos de obstrucción intestinal de 48 horas de evo-lución, es preferible la reducción quirúrgica.

4) Una invaginación asociada a divertículo de Meckel suele reducirse satisfactoriamente con enema opaco.

5) La mortalidad es muy elevada, aunque la reducción tenga lugar en las primeras 24 horas.

55. Niño de 4 meses que es llevado a consulta por estreñimiento crónico desde el nacimiento. Entre sus antecedentes personales destaca tapón meconial al nacimiento que se resolvió con enemas. Alimentación materna exclusiva. Talla y peso < P3. ¿Cuál de los siguientes datos NO apoyaría el diagnóstico?

1) Biopsia rectal patológica.2) Heces acintadas.3) Ampolla sin heces en el tacto rectal.4) Relajación del esfínter anal interno.5) Permanencia del bario en colon durante más de 24 horas al

realizar un tránsito digestivo.

56. Paciente de 12 meses que, de forma progresiva, inicia un cuadro de inapetencia, mal carácter, retraso del crecimiento ponde-roestatural, así como diarrea abundante, fétida y de aspecto grasiento. Exploración física: afebril, regular estado general, palidez cutaneomucosa, pérdida de masa muscular proximal y abdomen distendido. Señale lo FALSO respecto a la enfermedad celíaca:

1) Existe una predisposición genética, relacionada con los HLA B8, DR7, DR3 y DQ2.

2) Hay una mayor prevalencia de la enfermedad en niños con défi cit selectivo de IgA o diabetes mellitus.

3) El período más común de presentación es entre los 6 meses y los 2 años de edad.

4) Los anticuerpos antiendomisio IgA tienen una alta especi-fi cidad y sensibilidad.

5) El trigo, arroz, cebada y centeno deben ser excluidos de la dieta para toda la vida.

Page 8: Testcom1v Pd

Test 1.ª vuelta Distancia

CTO Medicina • C/Francisco Silvela, 106 • 28002 - Madrid • Tfno. (0034) 91 782 43 30/33/34 • E-mail: [email protected] • www. ctomedicina.com 8

Pediatríatiempos de protrombina y de tromboplastina parcial activada son normales. Ante este cuadro clínico, una de las siguientes afi rmaciones es cierta:

1) El diagnóstico más probable es el de púrpura trombótica trombocitopénica.

2) Requiere de transfusión urgente de concentración de pla-quetas.

3) La evolución más probable es a la recuperación espontánea.4) Debe practicarse una TC craneal para descartar hemorragia

intracraneal.5) El pronóstico depende de la precocidad del tratamiento.

66. Los tumores malignos son la principal causa de muerte entre el año y los 15 años de edad en los países desarrollados. Señale la afirmación INCORRECTA respecto a su epidemio-logía:

1) La leucemia linfoblástica aguda es la neoplasia maligna más

frecuente en la infancia, y muestra un ligero predominio en varones.

2) La incidencia más alta de enfermedad de Hodgkin se da en adolescentes y adultos jóvenes.

3) El neuroblastoma es el tumor sólido más frecuente en la infancia, fuera del sistema nervioso central.

4) El sarcoma de Ewing es el tumor óseo primario maligno más frecuente a partir de los 10 años, seguido del osteo-sarcoma.

5) El rabdomiosarcoma es el sarcoma de tejidos blandos más frecuente en la infancia, siendo la cabeza y el cuello las principales localizaciones.

67. Niña de 2 años presenta movimientos mioclónicos de extremi-dades y sacudidas desordenadas de los ojos. En la exploración se palpa masa abdominal en línea media y fl anco derecho. Se le realiza una TC abdominal, evidenciándose una masa de consis-tencia mixta y calcifi caciones en su interior. Señale la afi rmación falsa:

1) Es el tumor extracraneal más frecuente en la infancia.2) La edad más habitual al diagnóstico son los 2 años.3) La presentación con un síndrome de mioclonus- opsoclonus

ensombrece el pronóstico.4) Las catecolaminas en orina estarán elevadas.5) Se asocia con deleción del cromosoma 1 y anomalías del 17.

68. Señale la opción FALSA respecto al tumor de Wilms:

1) Se asocia a deleción del cromosoma 11, ya sea en las células tumorales o en todas las células del organismo.

2) Se asocia a aniridia y anomalías genitourinarias.3) La HTA es un hallazgo frecuente.4) El diagnóstico defi nitivo nos lo da la biopsia.5) Suele tratarse de una masa abdominal palpable asintomática,

con o sin metástasis pulmonares.

69. Un niño de 5 años, previamente sano, debuta bruscamente con palidez, astenia y anorexia; se le practica un hemograma y se evidencia anemia, leucopenia y trombopenia. En la exploración se aprecia hepatoesplenomegalia. ¿Qué diagnóstico le parece más probable?

61. Lactante de 3 meses consulta por cuadro de vómitos y febrícula de 48 horas de evolución, con progresiva pérdida del apetito. En la exploración no se objetiva foco infeccioso. Hemograma con leucocitosis y desviación a la izquierda. Sistemático de orina: leucocitos y nitritos positivos. ¿Cuál sería la actitud más adecuada?

1) Tratamiento con antitérmicos y ver evolución.2) Realizar punción lumbar.3) Diagnóstico de infección de orina y administrar antibió ticos

v.o.4) Realizar punción suprapúbica e iniciar antibióticos.5) Descartar refl ujo gastroesofágico.

62. Es FALSO, respecto al refl ujo vesicoureteral en el niño:

1) Puede sospecharse intraútero al ver dilatación pielocalicial en la ecografía obstétrica.

2) Suele dar lugar a infecciones urinarias de repetición.3) La indicación quirúrgica parte del número anual de recidivas

de infección urinaria.4) Puede ser útil hacer profi laxis antibiótica si se han repetido

varias infecciones urinarias.5) Si el refl ujo es masivo, puede llevar a IRC en la infancia y

puede requerir cirugía.

63. Varón de 2 años en cuya historia clínica destaca haber padecido un proceso diarreico en los días previos. En la exploración se observa palidez, estupor y hematuria. Ha presentado además una convulsión focal. Respecto al cuadro que usted sospecha, señale cuál de las siguientes opciones es FALSA:

1) Es la causa más frecuente de IRA en los niños pequeños.2) Existe anemia hemolítica microangiopática y trombopenia

dentro del cuadro clínico.3) La trombosis bilateral de las venas renales puede provocar

un cuadro similar.4) El empleo de corticoides supone el tratamiento de elección.5) La aparición de recidivas es muy poco frecuente.

64. Varón de 5 años que presenta de forma súbita dolor en teste derecho, sin antecedente de traumatismo previo. Exploración: afebril, teste tumefacto, doloroso a la exploración, con ausencia del refl ejo cremastérico. Señale la afi rmación FALSA:

1) En la eco-Doppler se observaría disminución del fl ujo san-guíneo en ese testículo.

2) El tratamiento consiste en antibióticos, antiinfl amatorios, y tras la fase aguda, cirugía.

3) La mayor incidencia se produce en la niñez tardía y adoles-cencia temprana.

4) En mayores de 13 años se debe hacer diagnóstico diferencial con una epididimitis.

5) En el periodo neonatal, generalmente tiene mal pronós tico.

65. Un niño de 2 años es traído a Urgencias por petequias generali-zadas sin otros signos de diátesis hemorrágica. Refi eren catarro de vías altas y fi ebre 2 semanas antes pero en la actualidad de encuentra afebril, con buen estado general, y el resto de la exploración es normal. Un hemograma muestra hemoglobina 14 mg/dl, leucocitos 9.400/mm3 y plaquetas 34.000 mm3. Los

Page 9: Testcom1v Pd

Test 1.ª vuelta Distancia

Pediatría

CTO Medicina • C/Francisco Silvela, 106 • 28002 - Madrid • Tfno. (0034) 91 782 43 30/33/34 • E-mail: [email protected] • www. ctomedicina.com 9

plaquetas 71.000/ mm3. De momento no dispone de frotis de sangre periférica, ni de reactantes de fase aguda, pero ya tiene una sospecha diagnóstica. Señale cuál es:

1) Mononucleosis infecciosa.2) Leucemia linfoblástica aguda.3) Linfoma en fase leucémica.4) Hemoglobinuria paroxística nocturna (HPN).5) Aplasia medular por antibióticos.

74. Un niño de 4 años acude al pediatra por aparición de lesiones papulosas rojas, algunas con vesículas blanquecinas no umbi-licadas en tronco y mucosa oral, muy pruriginosas. Dos días antes presentaba cuadro catarral con fi ebre moderada. Respecto al cuadro que presenta este niño, todo lo siguiente es cierto, EXCEPTO:

1) Está causado por un poxvirus, el virus de la varicela zóster.2) Las lesiones costrosas no contienen virus viables.3) La complicación más frecuente es la sobreinfección de las

lesiones cutáneas.4) La neumonía varicelosa es poco frecuente en la infancia.5) La encefalitis postvaricela que se presenta con signos cere-

belosos tiene mejor pronóstico que si lo hace con signos cerebrales.

75. Niño de 7 años presenta rash eritematoso en ambas mejillas, tronco y zona proximal de extremidades, con tendencia a acla-rarse en la zona central. Con respecto a esta enfermedad, señale la afi rmación FALSA:

1) Es producida por el parvovirus B19.2) El período de incubación es de 7 días.3) El período prodrómico se caracteriza por fi ebre alta.4) El exantema característico aparece en tres etapas.5) Como complicación puede producirse una crisis aplásica

grave.

76. Un lactante de 7 meses presenta súbitamente fi ebre de 40 ºC. No existe ningún otro síntoma y la exploración física es normal, salvo el hallazgo de una coriza ligera y adenopatías cervicales posteriores. El niño parece encontrarse bastante bien. ¿Cuál de las siguientes afi rmaciones describe mejor los factores diagnós-ticos que deben considerarse en este caso?

1) Si apareciera una erupción cutánea 24 horas después de comenzar la fi ebre, sería probable el diag nóstico de roséola o exantema súbito.

2) En la roséola infantil, el diagnóstico diferencial puede ser difícil, puesto que en las primeras 36 horas cursa con leu-cocitosis, con predominio de neutrófi los.

3) La fi ebre podría corresponder a los pródromos de una rubéola.

4) Si la fi ebre dura 3 días, desaparece luego rápidamente y aparece una erupción cutánea en ese momento, sería pro-bable el diagnóstico de eritema infeccioso.

5) Es probable una escarlatina, pues es una enfermedad propia de niños de 6 a 18 meses.

77. Jennifer tiene 16 meses de edad y acude a Urgencias por pre-sentar desde hace 2 días fi ebre de 39 ºC que su pediatra atribuyó

1) Anemia ferropénica.2) Linfoma no Hodgkin.3) Aplasia medular adquirida.4) Aplasia medular congénita.5) Leucemia aguda.

70. Con respecto al tumor de Wilms, es FALSO que:

1) Puede cursar con poliglobulia.2) En la UIV se observa una masa dependiente del tejido renal.3) La cirugía se indica, aun en presencia de metástasis pulmo-

nares.4) Posee gran tendencia a presentar calcifi caciones intraneoplá-

sicas.5) Los tumores bilaterales son más frecuentes en los casos

familiares.

71. Niña de 6 años que presenta desde hace 3 días fi ebre de 38 ºC que no cede con antitérmicos, tos seca y facies congestiva. Hoy acude por aparecer exantema maculopapuloso, rojo intenso, no puntiforme, confl uente, que no se blanquea a la presión en la parte superior del tórax, cara y parte proximal de EE.SS. A la exploración: Tª 40,3 ºC, adenopatías bilaterales en ángulo mandibular y lesiones blanquecinas sobre base eritematosa en mucosa subyugal. Respecto a la enfermedad que padece esta paciente, señale la afi rmación FALSA:

1) El exantema comienza por la cara, tiene evolución descen-dente, y desaparece en el mismo orden en el que apareció.

2) La neumonía de células gigantes de Hecht es menos fre-cuente que la neumonía por sobreinfección bacteriana.

3) La gravedad de la enfermedad está directamente relacionada con la intensidad y confl uencia del exantema.

4) El prurito suele ser intenso.5) Entre las posibles complicaciones se encuentra la anergia

cutánea y la reactivación de una TBC preexistente.

72. Niña de 5 años presenta cuadro catarral con fi ebre de 37,5 ºC. A la exploración presenta adenopatías retroauriculares y cervicales dolorosas a la palpación e hiperemia conjuntival. 48 horas después, aparece un exantema morbiliforme en cara. ¿Cuál de las siguientes afi rmaciones sobre la epidemiología de esta enfermedad es cierta?

1) Las personas con enfermedad subclínica no son contagiosas.2) Los niños se afectan con mayor frecuencia que las niñas.3) Los anticuerpos maternos protegen los 6 primeros meses

de vida.4) Un solo ataque no suele conferir inmunidad de duración

permanente.5) La relación de enfermedad no aparente/manifi esta es de 1

a 1.

73. Niño de 11 años, que consulta por fi ebre, dolor de garganta y cansancio, de 5 días de evolución. En la exploración física destacan unas amígdalas hipertrófi cas con exudado en sábana, adenopatías dolorosas axilares, inguinales, y las más grandes, las cervicales, y hepatoesplenomegalia de 3-4 cm por debajo de los rebordes costales. Pide un análisis de sangre, y lo pri-mero que le mandan del laboratorio es el hemograma: Hb 8,1 g/dl; Hto 27%; leucocitos totales 1.800/ mm3 (600 neutrófi los);

Page 10: Testcom1v Pd

Test 1.ª vuelta Distancia

CTO Medicina • C/Francisco Silvela, 106 • 28002 - Madrid • Tfno. (0034) 91 782 43 30/33/34 • E-mail: [email protected] • www. ctomedicina.com 10

Pediatría3) La neumonía por MAI es la infección oportunista más fre-

cuente en el SIDA pediátrico.4) La manifestación más frecuente en el SIDA infantil es la

hepatitis B.5) La parotiditis es más frecuente en el SIDA infantil que en el

del adulto.

82. Sobre el diagnóstico de un niño VIH positivo, es FALSO que:

1) Una IgG+ en un lactante de 10 meses indica infección activa por el VIH.

2) La mayoría de los niños infectados presenta hipergamma-globulinemia policlonal precoz.

3) La linfopenia e inversión del cociente CD4/CD8 son menos llamativas que en el adulto.

4) No es útil la determinación de IgA contra el VIH, porque no suele aparecer hasta el 6.º mes de vida.

5) Los resultados positivos en la PCR han mostrado una corre-lación positiva con el aislamiento del virus en cultivo.

83. Respecto a la vacuna triple vírica, señale la afi rmación FALSA:

1) Es una vacuna de microorganismos atenuados.2) La alergia al huevo ha dejado de ser una contraindicación

absoluta para su administración.3) Está contraindicada su utilización en embarazadas y

pacientes VIH positivos.4) Su administración en el calendario actual se realiza a los 15

meses y a los 4 años.5) En zonas de elevada morbimortalidad puede administrarse

la primera dosis a los 6-12 meses, con una segunda dosis de refuerzo después del año de vida.

84. Señale la afi rmación FALSA respecto a la vacuna de la polio:

1) La vacuna tipo Sabin es de virus vivos atenuados.2) La vacuna oral produce inmunidad local (IgA) y general (IgG).3) En el calendario actual se administra a los 2-4-6-18 meses y

entre los 4 y 6 años.4) La vacuna tipo Salk está contraindicada en familiares de

inmunodeprimidos.5) En caso de diarrea o vómitos sería conveniente retrasar la

vacunación con virus atenuados.

85. Señale la afi rmación cierta respecto a la vacuna DTP:

1) La vacuna de la tos ferina está contraindicada en mayores de 18 meses.

2) No puede administrarse conjuntamente con otras vacunas.3) La vacuna DTPa (acelular) tiene mayor riesgo de efectos

secundarios que la DTP, por lo que no se utiliza de forma rutinaria.

4) A los 14 años es preciso vacunar con Td (tétanos y difteria adultos), y no son necesarias nuevas dosis de recuerdo.

5) No está contraindicada en pacientes inmunodeprimidos.

86. Con respecto a la vacuna contra Haemophilus infl uenzae tipo B, es FALSO que:

1) Las nuevas vacunas conjugadas son aptas para la inmuni-zación en lactantes mayores de 2 meses.

a cuadro respiratorio. A la exploración presenta fi ebre de 40 ºC y contractura antiálgica en fl exión de extremidad inferior izquierda. Queda ingresada para estudio, y 4 días más tarde persiste el cuadro febril, junto con edema indurado en ambos pies, fi suras labiales, conjuntivitis no purulenta y exantema polimorfo en tronco. El diagnóstico más probable será:

1) Rubéola.2) Mononucleosis infecciosa.3) Púrpura de Schönlein-Henoch.4) Enfermedad de Kawasaki.5) Eritema infeccioso.

78. Paciente varón de 5 años de edad que acude por presentar cuadro febril y odinofagia con exudado blanquecino en ambas amígdalas que su pediatra trató con penicilina oral. Dos días después, presenta petequias y púrpura palpables, principal-mente en miembros inferiores, así como artralgias en ambos tobillos que le impiden la marcha y dolor abdominal de tipo cólico. Los estudios complementarios revelan 13 g/dl de Hb; leucocitos 10.500/mm3 con fórmula normal; plaquetas 485.000/mm3; tiempo de protrombina de 95% y tiempo parcial de trom-boplastina de 27 segundos (control 25 segundos). La causa más probable de estos hallazgos es:

1) Reacción a la penicilina.2) Púrpura de Schönlein-Henoch.3) Mononucleosis infecciosa.4) Enfermedad de Kawasaki.5) Púrpura trombocitopénica idiopática.

79. Respecto a la mononucleosis infecciosa, es FALSO que:

1) Hasta en el 80% hay aumento de transaminasas.2) En la analítica aparece leucopenia.3) Títulos elevados de anticuerpos frente al virus de Epstein-

Barr.4) Existe un síndrome linfoproliferativo en varones que han

sufrido la infección por este virus.5) La complicación más temida es la rotura esplénica.

80. Un bebé de 3 meses, que recibe lactancia materna, presenta una historia de accesos de tos con congestión facial, lagrimeo y acaba vomitando. Hace 2 semanas presentó cuadro catarral de vías áreas altas. En la Rx tórax se aprecian breves infi ltrados perihiliares. Con respecto a este cuadro, señale la respuesta que NO sea cierta:

1) Puede aparecer al principio de la vida por falta de anticuerpos maternos.

2) La fase catarral precede al período paroxístico.3) La tos paroxística puede durar de 1 a 4 semanas.4) El recuento de leucocitos suele elevarse en forma conside-

rable con predominio de polimorfonucleares.5) El período de incubación oscila entre 5 y 10 días.

81. Con respecto al SIDA en la infancia, señale la opción correcta:

1) El SIDA precoz es menos frecuente que el tardío y la clínica predominante es infecciosa.

2) El SIDA tardío es más frecuente y predomina la clínica neu-rológica.

Page 11: Testcom1v Pd

Test 1.ª vuelta Distancia

Pediatría

CTO Medicina • C/Francisco Silvela, 106 • 28002 - Madrid • Tfno. (0034) 91 782 43 30/33/34 • E-mail: [email protected] • www. ctomedicina.com 11

92. Ante un recién nacido de 2 días con cianosis, taquipnea sin soplo a la auscultación y que, en la radiografía de tórax, muestra ligera cardiomegalia, con un pedículo cardíaco estrecho y aumento del flujo sanguíneo pulmonar, el diag-nóstico más probable es:

1) Tetralogía de Fallot.2) Coartación de aorta.3) Anomalía de Ebstein.4) Transposición simple de las grandes arterias.5) Atresia tricuspídea.

93. Lactante de 9 meses de edad que llega a Urgencias traído por su madre, por somnolencia de 12 horas de evolución. No ha tenido fi ebre, vómitos, ni ninguna otra sintomatología. En la actitud de la madre destaca que parece darle poca importancia al estado del niño, contestando a las preguntas de la anam-nesis con desgana, aunque espontáneamente expresa todas las difi cultades por las que está pasando para compaginar su papel de madre con su trabajo. En la exploración física, el niño está consciente y reacciona con irritabilidad a estímulos, pero tiende a quedarse dormido. Tiene un hematoma periorbitario reciente, y varios pequeños antiguos en la región dorsolumbar, que la madre atañe a caídas accidentales, aunque el niño está iniciando ahora la bipedestación con ayuda. ¿Qué sospecharía inicialmente como causante del estado del niño?

1) Meningitis bacteriana.2) Malos tratos.3) Enfermedad metabólica no diagnosticada descompensada.4) Hipoglucemia.5) Hiponatremia.

94. Con respecto a la coartación de aorta (CoA), señale la FALSA:

1) El tipo más frecuente es la coartación de localización yux-taductal.

2) La mayor parte de los casos se muestran asintomáticos.3) La tensión arterial en los miembros inferiores es mayor que

en los superiores.4) En ocasiones, la tensión arterial en el brazo derecho es mayor

que en el izquierdo.5) En la radiografía de tórax, con el tiempo, se visualizan esco-

taduras en los bordes inferiores de las costillas.

95. Con respecto al desarrollo puberal normal, es verdadero:

1) El primer dato a la exploración sugestivo de inicio de la pubertad en las niñas es la pubarquia.

2) El aumento del tamaño testicular en el varón es el primer signo de pubertad.

3) La pubertad suele iniciarse a los 8 años en las niñas y a los 9 en los niños.

4) El vello sexual es el primer signo de pubertad.5) La ganancia de peso y talla es más acusada en las niñas que

en los niños.

96. Niña de 4 años que consulta por desarrollo mamario lentamente progresivo desde hace 2 meses, sin adrenarquia ni aumento de la velocidad de crecimento. Los padres no lo relacionan con ningún desencadenante ni con otros signos-síntomas. La exploración

2) Es muy inmunógena.3) Sus efectos adversos son mínimos.4) Es necesario aplicarla después de los 5 años.5) En el calendario vacunal debería administrarse a los 2-4-6 y

18 meses.

87. Señale la afi rmación FALSA respecto a la vacuna del VHB:

1) La vacuna de VHB está realizada por ingeniería genética con efectos adversos mínimos.

2) La pauta de administración en caso de riesgo inmediato como contacto con una jeringuilla sería 0-1-2 meses, y al año.

3) La vacuna VHB no está recomendada en mujeres embara-zadas.

4) No está indicada su administración en niños mayores de 7 años.

5) Se puede administrar con la gammaglobulina anti-VHB, siempre que se administre en distinto punto.

88. ¿Cuál sería la actitud más adecuada a seguir en los contactos de un niño con meningitis meningocócica?

1) Realizar quimioprofi laxis con ceftriaxona durante 2 días.2) No existe quimioprofi laxis efi caz.3) Administración de la vacuna antimeningocócica en niños

menores de 5 años.4) Administración de rifampicina durante 2 días.5) Administración de rifampicina durante 4 días.

89. ¿Cuál sería la actitud a seguir ante un niño de 4 años asintomático con Mantoux negativo, hijo de un tuberculoso bacilífero?

1) Observación y repetir Mantoux en 6 meses.2) Realizar quimioprofi laxis primaria durante 2 meses con INH

y repetir el Mantoux posteriormente.3) Realizar quimioprofilaxis primaria durante 6 meses con

INH.4) Administrar tratamiento completo con tres fármacos durante

6 meses.5) Vacunación con BCG.

90. Respecto al síndrome de muerte súbita infantil o del lactante (SMSL), señale cuál de los siguientes NO constituye un factor de riesgo:

1) Antecedentes familiares.2) Prematuridad.3) Lactancia materna.4) El sexo masculino.5) El tabaquismo materno.

91. La persistencia del ductus arterioso se asocia a todo lo siguiente, EXCEPTO a:

1) Pulsos periféricos saltones.2) Mayor frecuencia en prematuros.3) Soplo continuo en el área pulmonar.4) Puede ser efectivo el tratamiento con indometacina.5) Las prostaglandinas son el mejor tratamiento médico para

conseguir su cierre.

Page 12: Testcom1v Pd

Test 1.ª vuelta Distancia

CTO Medicina • C/Francisco Silvela, 106 • 28002 - Madrid • Tfno. (0034) 91 782 43 30/33/34 • E-mail: [email protected] • www. ctomedicina.com 12

Pediatríafísica general es normal, con un estadio puberal II de Tanner (T2 P1 Aa). Se realiza una radiografía de mano y muñeca izquierdas, estimándose la edad ósea en 3,5-4 años. En la ecografía pélvica se visualizan útero y ovarios de morfología y tamaño adecuados a la edad de la paciente, sin quistes signifi cativos. Los niveles séricos de estradiol son de 40 pg/dl (N 10-30), y la FSH y la LH basales son normales. ¿Cuál es su diagnóstico de sospecha?

1) Pubertad precoz central, probablemente idiopática.2) Pubertad precoz central, probablemente por hamartoma

hipotalámico.3) Telarquia prematura aislada.4) Pubertad precoz periférica.5) Aunque no sepamos el índice de masa corporal (IMC) de la

paciente, lo más probable es que sea elevado y nos encon-tremos realmente ante una adipomastia bilateral.

97. ¿Cuál de los siguientes cuadros no forma parte de diagnóstico diferencial de talla baja en los niños?

1) Síndrome de Turner.2) Síndrome de Down3) Síndrome de Silver-Russell.4) Síndrome de Klinefelter.5) Síndrome de Noonan.

98. Respecto a las convulsiones neonatales, señale la afi rmación FALSA:

1) Los RN con convulsiones suelen tener mal pronóstico.2) El tratamiento consiste en medidas de sostén y fenobarbital

o diazepam.3) Las crisis tónico-clónicas generalizadas son raras durante el

primer mes de vida.4) La clínica es variable y el EEG puede ser la única forma para

su diagnóstico.5) La causa más frecuente de crisis en el período neonatal es

la encefalopatía hipoxicoisquémica.

99. Acerca de la enuresis, señale cuál es la respuesta correcta:

1) Es más frecuente en el sexo femenino.2) La enuresis secundaria es más frecuente entre los 5 y los 8

años de edad, y tiene peor pronóstico que la primaria.3) La predisposición genética es un factor de riesgo muy

importante para la enuresis nocturna.4) La causa más frecuente de enuresis diurna es la infección

de orina.5) El tratamiento de elección en la enuresis nocturna persistente

es la desmopresina administrada antes de acostarse.

100. Llega a Urgencias un niño de 18 meses con fi ebre de 39 ºC, que ha presentado en su domicilio un episodio tónico-clónico gene-ralizado de 3 minutos de duración, con posterior somnolencia. Lo correcto a su llegada a Urgencias sería:

1) Administración de bolo de glucosa.2) Administración de antitérmicos y diazepam.3) Administración de fenitoína.4) Realizar punción lumbar.5) Administración de piridoxina.

Page 13: Testcom1v Pd

Comentarios de Test a distancia 1.ª vuelta

CTO Medicina • C/Francisco Silvela, 106 • 28002 - Madrid • Tfno. (0034) 91 782 43 30/33/34 • E-mail: [email protected] • www. ctomedicina.com 1

Pediatría

Comentarios de Test a distancia 1.ª vuelta

Pediatría

Pregunta 1.- R: 3El test de Apgar se debe realizar a todo recién nacido, independientemente de su edad gestacional sano o con sospecha de patología (respuesta 1 falsa). Se realiza al minuto y a los 5 minutos y posteriormente sólo se repetirá cada 5 minutos si la puntuación a los 5 minutos es inferior de 7 (respuesta 3 correcta).Entre los parámetros que valora se encuentran: color, frecuencia cardíaca, respuesta a la estimulación, tono muscular y esfuerzo respiratorio. NO frecuencia respiratoria (respuesta 2 falsa).A pesar de una puntuación baja en los primeros minutos de vida, no sirve para establecer valoraciones pronósticas (respuesta 4 falsa). Una puntuación inferior a 3, mantenida más allá de los 20 minutos de vida, sí puede predecir una elevada morbimortalidad.La palidez cutánea generalizada supone una puntuación de 0 en el test de Apgar (respuesta 5 falsa).

Pregunta 2.- R: 4Si seguimos la tabla del comentario de la pregunta anterior obtendremos una puntuación de 4 en el test de Apgar realizado a este recién nacido.

Pregunta 3.- R: 1La alimentación de los prematuros debe realizarse de forma cautelosa. Siempre que no haya distrés respiratorio ni otras circunstancias que contraindiquen su inicio, hay que procurar una alimentación precoz para evitar la hipoglucemia, la deshidratación y la hiperbilirrubinemia. Se puede intentar vía oral y continuar si hay buena tolerancia, aunque en los menores de 32 semanas es muy frecuente que no exista la coordinación adecuada y haya que colocar una SNG (respuesta 1 falsa).

La anemia fi siológica, aparece antes y con más intensidad que en los nacidos a término, por los menores depósitos de hierro y el rápido crecimiento, por lo que a veces precisan ser tratados con eritropoyeti-na, que debe acompañarse siempre de suplementos de hierro vía oral (respuesta 2 correcta).Las pérdidas insensibles de líquido están aumentadas respecto a los neonatos nacidos a término, en relación inversa a la edad gestacional, y sus necesidades diarias son mayores (respuesta 3 correcta).En todos los recién nacidos, pero más en los prematuros, los tóxicos de eliminación renal, y en los prematuros también los de eliminación hepática, deben darse en menor dosis o con intervalos mayores (res-puesta 4 correcta).En lo RNPT, la hipoxia produce afectación de la matriz germinal, estructura transitoria presente en el cerebro inmaduro hasta la semana 34 de gesta-ción. Dado que está compuesta por vasos su lesión produce hemorragia. Hay que realizar sistemáticamente ecografía craneal a todos los recién nacidos < 32 SG y a los recién nacidos > 32SG con factores de riesgo como asfi xia perinatal o signos neurológicos anormales (respuesta 5 correcta).

Pregunta 4.- R: 3La respuesta 1 es cierta: es importante para el MIR tener claro las diferencias entre caput (edema de partes blandas, no limitado por suturas) y cefa-lohematoma (hematoma entre hueso y periostio, limitada por suturas).La respuesta 2 es cierta: el traumatismo obstétrico más frecuente es la fractura de clavícula. A la exploración lo más llamativo es la crepitación y el refl ejo de Moro asimétrico.La respuesta 3 es falsa: el llamado nódulo de Stromayer es un hematoma palpable a nivel del ECM. La mayoría de las veces se reabsorbe espontá-

0 1 2

Esfuerzo respiratorio Ausente Lento, irregular Bueno, llanto

Respuesta a la introducción de una sonda nasogástrica

Sin respuesta Mueca Tos o estornudo

Frecuencia cardíaca Ausente Menos de 100 Más de 100

Tono muscular

DébilLigera fl exión

de extremidadesMovimientos activos

Color Azul, pálidoCuerpo sonrosado,

extremidades cianóticasTotalmente sonrosado

Pregunta 1. Test de Apgar.

Page 14: Testcom1v Pd

Comentarios de Test a distancia 1.ª vuelta

CTO Medicina • C/Francisco Silvela, 106 • 28002 - Madrid • Tfno. (0034) 91 782 43 30/33/34 • E-mail: [email protected] • www. ctomedicina.com 2

Pediatríaneamente, pero en ocasiones se puede fi brosar acortando el músculo. El tratamiento se basa en la fi sioterapia y muy rara vez requiere cirugía.La respuesta 4 es cierta: la víscera que con mayor frecuencia se lesiona es el hígado, pero la mayoría de las veces son pequeños hematomas sub-capsulares limitados que, secundariamente, pueden producir ictericia precoz prolongada, sin otra repercusión clínica. Recordar para el MIR el cuadro clínico típico de la hemorragia suprarrenal: niño grande, parto de nalgas, que en las primeras horas de vida presenta clínica de hipotensión, sangrado y shock.La respuesta 5 es cierta: las fracturas craneales lineales generalmente no precisan tratamiento quirúrgico, salvo que se traten de fracturas deprimidas que produzcan lesiones en el parénquima.

Pregunta 5.- R: 4Dentro de los fenómenos dermatológicos que podemos encontrar en un neonato, y que no tienen signifi cación patológica, se hallan: los quistes de milium, la mancha mongólica, los angiomas planos, el eritema tóxico y la melanosis pustulosa.El eritema tóxico es más propio de la raza blanca y consiste en vesiculopús-tulas sobre base eritematosa que suele respetar palmas y plantas y tiende a desaparecer en la primera semana. La melanosis pustulosa es más frecuente en la raza negra, la erupción vesiculopustulosa sí suele afectar a las palmas y plantas, y tiene tendencia a desaparecer en varias semanas. Recuerda las principales diferencias entre eritema tóxico y la melanosis pustulosa.

ERITEMA TÓXICO MELANOSIS PUSTULOSA

Aparición 1-3 días Nacimiento

Localización· Variable· No palmoplantar

· Variable· Sí palmoplantar

Frotis Eosinófi los Neutrófi los

Cultivo Estéril Estéril

Pregunta 5. Melanosis pustulosa vs. eritema tóxico del RN.

Las manchas hipopigmentadas geográfi cas se asocian a esclerosis tuberosa (respuesta 1).La ictericia en las primeras 24 horas de vida es siempre patológica (res-puesta 2) por hemólisis e infecciones (sepsis, TORCH).El nevus fl ammeus o mancha “en vino de Oporto” se asocia a angiomatosis encefalotrigeminal o síndrome de Sturge-Weber (respuesta 3).Manchas café con leche se asocian a neurofi bromatosis tipo 1 (res-puesta 5).

Pregunta 6.- R: 2 El cuadro clínico se refi ere a una taquipnea transitoria del RN, síndrome de Avery, pulmón húmedo o SDR tipo II.La presentación típica es un RN nacido por cesárea o por parto vaginal rápido, lo que supone que el tórax no ha estado sometido a la presión positiva que exprime de líquido los alveolos del pulmón. La clínica se inicia en las primeras horas de vida, que cede y mejora con medidas poco agresivas (oxígeno en incubadora). Suelen alcanzar la resolución clínica en un plazo máximo de 3 días.Lo que establece el diagnóstico, aparte de la evolución clínica, es la Rx de tórax, donde se aprecia aumento de marcas vasculares, líquido en las cisuras. La EMH es típica de prematuros, especialmente habitual en hijos de madre diabética, por défi cit de surfactante y por inmadurez estructural pulmonar. Rx de tórax: patrón reticulonodular (vidrio esmerilado) con broncograma aéreo (respuesta 1 falsa).El SAM es típico de RN postérmino, con antecedentes de estrés en el momento del parto. Rx tórax: infi ltrados parcheados algodonosos (respuesta 3 falsa).

La PCF suele cursar con clínica marcada de cianosis que no responde a la admi-nistración de oxígeno. No hay una Rx de tórax característica (respuesta 4 falsa).La DBP es típica de pretérminos que han necesitado oxígeno con el antecedente de barotrauma o EMH. Rx de tórax: patrón “en esponja”. (respuesta 5 falsa).

Pregunta 7.- R: 2Estamos ante un cuadro típico de enfermedad de membrana hialina: RNPT con difi cultad respiratoria intensa que aparece en las primeras horas de vida, que no mejora tras la administración de oxígeno. En la gasometría es llamativa la importante hipoxia. Para el diagnóstico es necesario realizar una Rx de tórax, donde lo más típico es encontrar un infi ltrado reticulono-dular (en vidrio esmerilado) con broncograma aéreo. La imagen radioló-gica no es patognomónica (también aparece en neumonías).Respuesta 3: corresponde a la taquipnea transitoria del RN.Respuesta 4: típico de la displasia broncopulmonar.Respuesta 5: corresponde a un neumotórax bilateral.

Pregunta 8.- R: 4El tratamiento de la EMH consiste en:• Monitorización estrecha, control de líquidos.• Intentar mantener una aceptable oxigenación (pO

2 50-70 mmHg). Si no

se consigue con oxígeno indirecto, será necesario intubar al paciente e iniciar ventilación mecánica.

• Administración de surfactante intratraqueal en las primeras 24 horas de vida, pudiendo recibir hasta 4 dosis según evolución. Esta medida se ha visto que mejora la clínica de la EMH, pero que no disminuye los casos de DBP.

• Administración de antibióticos: ampicilina más gentamicina, porque la clínica y la Rx pueden ser semejantes en el caso de una sepsis con neumonía.

La respuesta incorrecta es la 4. Las prostaglandinas en los neonatos están indicadas en caso de cardiopatías ductus dependientes, en las que es IMPRESCINDIBLE mantener el ductus abierto.

Pregunta 9.- R: 2El caso clínico es el de un niño que ha sufrido un SAM. Lo típico es que se produzcan en RN postérmino, con un estrés en el momento del parto, que estimula el peristaltismo intestinal y se produce la eliminación de meconio intraútero.Al nacimiento, el meconio se encuentra en las vías aéreas altas; si no se elimina a tiempo, éste pasará al pulmón con las primeras respiraciones, formando tapones que ejercen mecanismo valvular favoreciendo el atropamiento de aire y produciendo un cuadro de alveolitis con riesgo de atelectasias y sobreinfección bacteriana, siendo el germen más frecuente E. coli. En los casos más graves se asocia con una alta mortalidad debido a la hipertensión pulmonar que desarrollan.El tratamiento consiste en ventilación mecánica, antibióticos profi -lácticos y medidas para disminuir la hipertensión pulmonar: alcalinizar, tolazolina y oxido nítrico inhalado. Algunos niños se pueden benefi ciar de la ECMO (oxigenación por membrana extracorpórea).

Pregunta 10.- R: 1Caso clínico de una parálisis braquial superior, también conocida como pa-rálisis de Erb-Duchenne. Se debe a la afectación de las raíces C5-C6 del plexo braquial. El recién nacido presenta el brazo en aducción y rotación interna con el refl ejo de Moro ausente en el lado de la lesión (respuesta 1 correcta). Sin embargo, el refl ejo de prensión palmar suele estar presente al no verse afectadas las raíces inferiores. En los casos en que se lesione también la raíz C4 puede asociarse con una parálisis frénica.

Page 15: Testcom1v Pd

Comentarios de Test a distancia 1.ª vuelta

Pediatría

CTO Medicina • C/Francisco Silvela, 106 • 28002 - Madrid • Tfno. (0034) 91 782 43 30/33/34 • E-mail: [email protected] • www. ctomedicina. 3

PARÁLISIS BRAQUIAL P. ERB-DUCHENNE P. KLUMPKE

Raíces (C4) - C5 - C6 C7 - C8 - (T1)

ClínicaBrazo en aducción y rotación interna

Mano caída

R. Moro No presente o asimétrico Presente

R. prensión palmar Presente No presente

Asociaciones C4 - Parálisis frénica T1 - S. Horner

Pregunta 10. Parálisis braquial.

Pregunta 11.- R: 5Estamos ante un caso típico de displasia broncopulmonar: niño de un mes de vida con antecedente de prematuridad y enfermedad de membrana hialina que requiere altas concentraciones de oxígeno para mantener adecuadas saturaciones. Dentro de los factores de riesgo para este cuadro fi guran: la toxicidad del oxígeno en altas concentraciones, inmadurez pulmonar y barotrauma. El diagnóstico se establece por la historia clínica y la Rx de tórax, donde es típico encontrar el llamado patrón de esponja (áreas más claras que alternan con otras de mayor densidad). El tratamiento se basa en oxigenoterapia, restricción de líquidos y diu-réticos, broncodilatadores y corticoides (sólo en casos graves). Se ha demostrado que la vitamina A disminuye la incidencia de DBP.La mayoría de los niños suelen tener un curso favorable, alcanzando la normalidad de la función pulmonar hacia los 2 años de vida. Hay un pequeño porcentaje que evoluciona hacia un cuadro de hipertensión pulmonar persistente (respuesta 5 falsa). Las dos causas más frecuentes de muerte en estos niños son la ICC derecha y la bronquiolitis necrotizante.

Pregunta 12.- R: 3Se trata de un caso típico de hernia diafragmática congénita. La posterior o de Bochdaleck es el tipo de hernia diafragmática más frecuente, sobre todo izquierda. Consiste en un orifi cio grande en la parte posterior izquier-da del diafragma a través del que vísceras abdominales migran hacia el tórax, con lo que el abdomen aparece excavado (vacío de contenido) y hay un compromiso respiratorio grave secundario a la hipoplasia y compresión pulmonar. Es un cuadro severo que aboca a la instauración de hiperten-sión pulmonar que ya está presente en el nacimiento (respuesta 5 falsa).El diagnóstico en la mayoría de los casos se hace por ecografía prenatal. Cuando no es así, se realiza Rx de tórax al nacimiento donde adverti-remos la presencia de asas intestinales en tórax, y el mediastino estará desplazado por el efecto masa de éstas. Si tras la radiografía hay dudas con malformaciones pulmonares quísticas, hay que realizar una ecografía (respuesta 3 correcta).El cierre quirúrgico se hará a las 24-72 horas tras estabilizar la situación hemodinámica del paciente, primero durante 10-12 horas forzaremos una situación de alcalosis (el pH alcalino vasodilata las arterias pulmo-nares) para minimizar la hipertensión pulmonar (respuesta 2 falsa). Prácticamente todos tienen refl ujo gastroesofágico al nacer, y hasta un 20% necesita una funduplicatura (respuesta 4 falsa).La hernia diafragmática congénita es más frecuente en algunos síndromes congénitos, como las trisomías 21, 13 y 18. En un 20-30% de los casos hay otras alteraciones asociadas (respuesta 1 falsa) como malrotación intestinal (la más frecuente), cardiopatías y malformaciones vasculares.

Pregunta 13.- R: 3Estamos ante caso típico de hemorragia de la matriz germinal: RNPT con distrés respiratorio y que de forma brusca desarrolla hipotensión, bradicardia, cianosis, fontanela abombada y disminución del hematocrito, el diagnostico será de hemorragia de la matriz germinal.

La matriz germinal es una estructura transitoria, presente en el cerebro inmaduro en la cabeza del núcleo caudado, cerca de los ventrículos late-rales hasta la semana 34 de gestación y es muy sensible a la hipoxia. Dicha matriz está constituida por vasos y su lesión produce hemorragia que se diagnostica a través de una ecografía transfontanelar (respuesta 3 correcta).

Pregunta 14.- R: 4Estamos ante un RN con un cuadro de enterocolitis necrotizante (NEC). La presentación típica es en un RNPT que a la segunda semana de vida inicia cuadro de vómitos, distensión abdominal y deposiciones con sangre. Dentro de los factores de riesgo fi guran, aparte de la prematuridad, la administración precoz y con altas concentraciones de leche, situaciones de hipoxia y bajo gasto y todos aquellos cuadros que favorezcan isquemia intestinal. La prueba diagnóstica indicada sería una Rx de abdomen donde se apreciaría el dato más característico, la neumatosis intestinal.El tratamiento se debe instaurar ante la mínima sospecha, puesto que de eso va a depender el pronóstico. Recuerda que la lactancia materna es factor protector. Si el niño no tiene signos de perforación intestinal, el tratamiento será dieta absoluta y antibioterapia que cubra anaerobios y gramnegativos. El tratamiento será quirúrgico en caso de perforación intestinal o sepsis refractaria al tratamiento médico. Las complicaciones a largo plazo son la estenosis intestinal y el síndrome de intestino corto.

Pregunta 15.- R: 3El cuadro clínico se refi ere a un íleo meconial. La forma de presentación más habitual en los neonatos es como un cuadro de obstrucción intes-tinal congénita. El dato que apoyaría este diagnostico sería palpar unos cordones duros que siguen el marco cólico, puesto que el lugar donde con mayor frecuencia se impacta el meconio es la zona de íleon distal y colón proximal. El diagnóstico se hace mediante radiografía de abdomen con distensión de asas intestinales y patrón granular espumoso en los puntos de concentración meconial.Una vez establecido el diagnóstico es siempre obligatorio descartar una fi brosis quística (un 15% debutan como íleo meconial).El tratamiento, si el niño no tiene signos de perforación intestinal, se realiza mediante la administración de enemas hiperosmolares. Si con esta medida no se soluciona el problema o se producen complicaciones, sería necesario recurrir a la cirugía, realizando una resección de la zona de impactación y anastomosis terminoterminal.

Pregunta 16.- R: 3Respecto a las ictericias neonatales, es importante tener claro varios aspectos. • A continuación se refl ejan las causas más frecuentes de ictericia

según el momento de aparición: 1.as 24 horas Hemólisis.

Infecciones: sepsis, TORCH.

2.º-3.er día Fisiológica. Infecciones: sepsis, TORCH. Anemias hemolíticas.

4.º-7.º día Sepsis. TORCH. Obstrucción intestinal. Lactancia materna.

>1 mes Galactosemia, hipotiroidismo, lactancia materna, metabolopatías, ictericia obstructiva, Gilbert,

Crigler-Najjar.• Características de la ictericia fi siológica vs. no fi siológica:

- Inicio en las primeras 24 horas: SIEMPRE patológico.- Predominio de bilirrubina directa: SIEMPRE patológico.

Page 16: Testcom1v Pd

Comentarios de Test a distancia 1.ª vuelta

CTO Medicina • C/Francisco Silvela, 106 • 28002 - Madrid • Tfno. (0034) 91 782 43 30/33/34 • E-mail: [email protected] • www. ctomedicina.com 4

PediatríaLas cifras más bajas se producen a los 2 meses en los RNPT y a los 3 en los RNT (respuesta 1 falsa).El tratamiento consiste en suplementos de hierro a partir de los 2 meses en los RNPT, ya que antes los depósitos están llenos por la hemólisis de los glóbulos rojos (respuesta 3 correcta), y en casos seleccionados sería necesario la transfusión de concentrados de hematíes (respuesta 4 correcta).En los neonatos no se manifi esta la betatalasemia, porque la cadena beta de la hemoglobina se empieza a sintetizar a partir de los 6 meses (respuesta 5 correcta).

Pregunta 21.- R: 3Es muy importante recordar las situaciones en las que se debe adminis-trar gammaglobulina específi ca anti-D para prevenir la isoinmunización Rh. Es necesario inyectar la gammaglobulina anti-D a las 28 semanas de gestación y en las primeras 72 horas después del parto, aborto o amniocentesis, si se confi rma que el recién nacido es Rh positivo y la madre Rh negativo. La profi laxis se hará sólo si la gestante no ha sido previamente sensibilizada, es decir, sólo si el test de Coombs indirecto de la gestante es negativo (respuesta 3 falsa).

Pregunta 22.- R: 5La hipoglucemia neonatal es bastante frecuente en los hijos de madre diabética (75% de los hijos de madres diabéticas y 25% de los hijos de madres con diabetes gestacional). Se debe a que el HMD sintetiza ex-ceso de insulina para compensar el défi cit materno de esta hormona y el riesgo es proporcional a los niveles de glucosa en sangre de cordón. Recuerda que puede aparecer en las primeras 48 horas de vida, riesgo máximo entre las 3 y las 6 horas de vida y normalmente cursa de forma asintomática.El control durante el parto de la glucemia materna junto con el control de la glucemia venosa o capilar en el neonato y el inicio precoz de la ali-mentación oral (respuesta 5 falsa), disminuyen el riesgo de hipoglucemia neonatal. El tratamiento consiste en el aporte de glucosa i.v. mediante perfusión continua, evitando los bolos por el riesgo de hiperglucemia de rebote.

Pregunta 23.- R: 1El embarazo del hijo de madre diabética es considerado un embarazo de alto riesgo, debido al gran número de problemas que puede presentar el RN. Entre ellos fi guran:• Mayor mortalidad fetal y neonatal.• Polihidramnios.• Macrosomía con visceromegalia si la madre no tiene vasculopatía,

si la tiene, CIR.• Estenosis subaórtica con hipertrofi a septal asimétrica.• Riesgo de EMH: la insulina en altas concentraciones inhibe la síntesis

de surfactante.• Policitemia y sus consecuencias (ictericia y trombosis de la vena renal).• Mayor incidencia de malformaciones congénitas:

- Malformaciones más frecuentes: cardíacas.- Malformación digestiva más frecuente: colon izquierdo hipoplá-

sico.- Malformación más característica: agenesia lumbosacra.

• Alteraciones metabólicas:- Hipoglucemia: máxima en las 3-6 primeras horas. - Hipocalcemia.

Pregunta 24.- R: 3Recuerda las principales características de la sepsis PRECOZ, TARDÍA y NOSOCOMIAL.

• Otros datos de la ictericia no fi siológica:- Incremento mayor de 5 mg/24 horas.- Bilirrubina en sangre de cordón mayor de 3 mg/dl.- RNT > 12 y RNPT > 14.- Duración superior a 14 días.

Pregunta 17.- R: 3La ictericia por lactancia materna (LM) es la causa más frecuente de ictericia tardía. Su frecuencia es de aproximadamente 1/200 (respuesta 1 correcta).Comienza a manifestarse entre el 5.º y el 7.º día de vida alcanzando su máximo en la tercera semana (respuesta 2 correcta).Se debe a que en la leche materna existen unas sustancias (pregnanodiol, ácidos grasos de cadena larga) que inhiben la glucuronil-transferasa, dando como resultado un aumento de la bilirrubina a expensas de fracción indirecta (respuesta 4 correcta).En ocasiones, cuando existen dudas diagnósticas, se puede interrumpir la LM transitoriamente durante unos días, observándose un claro descenso en la bilirrubina, pero ésta no es indicación de suspender defi nitivamente la LM, puesto que suele ser moderada (valores inferiores a 15) y no se han descrito casos de kernicterus (respuesta 3 falsa).

Pregunta 18.- R: 5Para el MIR es importante saberse bien las diferencias entre la incom-patibilidad Rh y AB0.

Rh madre (-), hijo (+) AB0 madre (0)

Frecuencia Menor Mayor

Aparición Después del primer embarazo Primer embarazo

Gravedad · Más grave (ictericia,

anemia)· Hidrops

· Leve (ictericia, anemia)· No hidrops

Diagnóstico Coombs directo +; indirecto +Coombs directo +/-; indirecto +

Tratamiento

· Exanguinotransfusión/ fototerapia

· Prevencióncon gammaglobulina anti-D

Fototerapia y/o exanguinotransfusión

Pregunta 18. Diagnóstico diferencial de la isoinmunización del recién nacido.

Pregunta 19.- R: 4El caso clínico hace referencia a una enfermedad hemorrágica del RN. La causa es un défi cit de vitamina K que produce a su vez défi cit de factores k dependientes (respuesta 1 correcta), debido a que durante el emba-razo el paso transplacentario de vitamina K es escaso. La LM es pobre en vitamina K (respuesta 4 falsa) y en el intestino no hay bacterias que sinteticen esta vitamina. Todas estas circunstancias obligan a administrar vitamina K (1 mg i.m.) a todos los RN en las primeras 24 horas de vida.La clínica consiste en sangrado a distintos niveles en las primeras 24 horas (respuesta 2 correcta): umbilical, digestivo, nasal, etc.El tratamiento consiste en la administración de nuevas dosis de vitamina K o plasma fresco congelado (respuesta 5 correcta). Es importante recordar que los hijos de madres que durante el embarazo han tomado fenitoína o fenobarbital tienen más riesgo de padecer este trastorno (respuesta 3 correcta).

Pregunta 20.- R: 1La llamada anemia fi siológica se debe a un défi cit transitorio de EPO, que unido a la hemólisis de glóbulos rojos durante los primeros meses de vida, dejan al niño anémico (respuesta 2 correcta).

Page 17: Testcom1v Pd

Comentarios de Test a distancia 1.ª vuelta

Pediatría

CTO Medicina • C/Francisco Silvela, 106 • 28002 - Madrid • Tfno. (0034) 91 782 43 30/33/34 • E-mail: [email protected] • www. ctomedicina. 5

y calcifi caciones periventriculares (en el caso de la toxoplasmosis, éstas son difusas). La infección congénita por CMV es la infección viral congénita más fre-cuente (respuesta 1 falsa). El 90% de los recién nacidos están asintomáticos (respuesta 2 correcta), pero si tienen clínica pueden presentar ictericia, hepatoesplenomegalia, calcifi caciones, hepatitis,etc. La neumonitis es la forma más característica de infección postnatal.La secuela más frecuente es la hipoacusia neurosensorial, no de trans-misión (respuesta 3 falsa).La afectación ocular típica es la coriorretinitis (respuesta 4 falsa).Estos niños suelen desarrollar microcefalia evolutivamente (respuesta 5 falsa).

Pregunta 27.- R: 2Recuerda que si la infección de la gestante por toxoplasma ocurre en el primer trimestre, el RN puede presentar la tétrada de Sabin consistente

Pregunta 25.- R: 5La madre que puede contagiar una hepatitis B al RN es aquella que tiene el Ags +, (hepatitis crónica activa, hepatitis aguda o portadora). Si además el Age es positivo, el riesgo asciende hasta un 90%. El momento de mayor riesgo es el momento del parto.La mayoría de las veces, la enfermedad en el neonato es asintomática, pero tiene una alta probabilidad de evolucionar hacia una forma crónica y sufrir degeneración maligna.Para evitar este curso es fundamental realizar profi laxis con gammaglo-bulina específi ca en las 12 primeras horas de vida y primera dosis de vacuna. Si la profi laxis se realiza correctamente, podrá recibir lactancia materna. Poste-riormente hay que seguir la pauta de vacunación de VHB a los 2 y 6 meses.

Pregunta 26.- R: 2El caso clínico nos presenta a un RN con clínica de CMV congénita. Los datos claves que nos permiten llegar al diagnóstico son: coriorretinitis

IgGCTOMEGALOVIRUS TORMOPLAMA

Infección materna• Se reactiva asintomática 1-2%• Se infecta sintomática 1-2%0

• Se infecta

Transmisión• Placentaria 1.er trimestre más frecuente, más grave• Canal de parto• Lactancia

• Placentario1.er trimestre más grave3.er trimestre más frecuente

Ansiolítico más frecuente• 5-18% secuelas tardías, hipoacusia neurosensorial bilateral y severa.

Sin tratamiento coriorretinitis

Síntoma• Corriorretinitis• Microcefalia• Calcifi caciones periventriculares

SABIN• Coriorretinitis• Hidrocefalia• Calcifi caciones intracraneales difusas• Convulsiones

Screening prenatal • Prenatal Ac antiCMV (no se hace prevención)

1.ª determinación:• Negativo: vigilar• Positivo: hacer una 2.ª determinación:- Disminuido igual: infección pasada- Aumantado: tratar

Diagnóstico• Aislamiento y cultivo en orina “inclusiones en ojo de búho”• IgM• IgG o estable a las 6 semanas

• Aislamiento en placenta (¿sangre?)• IgM• IgG o estable a las 6 semanas

TratamientoNo• Ganciclovir a veces

Siempre: • Sintomático: primetadina + sulfadicina 6 meses primetadina + sulfamicina/espiromicina 6 meses• Asintomático: espiromicina y valorar serología

Pregunta 27. Diferencias entre toxoplasma y citomegalovirus.

ETIOLOGÍA CLÍNICA CURSO PRONÓSTICO TRATAMIENTO

Sepsis precoz (3.º a 5.º día)

· S. agalactiae· E. coli· Listeria monocytogenes

· Más afectación respiratoria (quejido, polipnea, tiraje, aleteo, cianosis), ictericia

· Listeria: además,granulomas en faringe, petequias

Fulminante

· Muy malo (mueren 30%)

· La listeriosis es muy grave (mueren 40-80%)

Ampicilina + gentamicina (ampicilina + cefotaxima, si meningitis)

Sepsis tardía (7-28 días)

· S. agalactiae serotipo III· E. coli serotipo K1

· Más afectación del SNC (meningitis neonatal)

· Focalizan más

Menos fulminante

Menos maloAlta morbilidad

· Ampicilina + gentamicina· Si meningitis:

- No ingresados: ampicilina+ cefa 3.º

Sepsis nosocomial

· S. epidermidis· S. aureus· P. aeruginosa· C. albicans

Variable Variable Variable

· Vancomicina + amikacina+ anfotericina B

· Si meningitis: vancomicina + ceftacidina+ anfotericina B

Pregunta 24. Sepsis neonatal.

Page 18: Testcom1v Pd

Comentarios de Test a distancia 1.ª vuelta

CTO Medicina • C/Francisco Silvela, 106 • 28002 - Madrid • Tfno. (0034) 91 782 43 30/33/34 • E-mail: [email protected] • www. ctomedicina.com 6

Pediatría• Al contrario que en todas las infecciones connatales, la rubéola tiene

más riesgo de trasmisión en el 1.er trimestre. También es más grave si se infecta en este trimestre.

• La tríada de Gregg se caracteriza por:- Hipoacusia neurosensorial: es la manifestación más habitual.- Cardiopatía: la más frecuente, el DAP.- Afectación ocular: lo más habitual son las cataratas.

• Un RN infectado por rubéola congénita puede eliminar el virus en las secreciones hasta año y medio después del nacimiento, por lo que es obligatorio aislarlo de mujeres embarazadas.

Pregunta 29.- R: 1De los factores que se citan, el más relacionado con el CIR tipo simétrico (CIR tipo I) es la infección congénita. Recuerda que hay que sospechar una infección connatal ante la presen-cia de CIR tipo simétrico, hepatoesplenomegalia, adenopatías, ictericia, anemia y trombopenia en un recién nacido.La causa más frecuente de CIR tipo asimétrico (CIR tipo II) es la insufi -ciencia placentaria.

Pregunta 30.- R: 4Todo el caso clínico hace referencia a la infección por el virus herpes simple, siendo el más frecuente el tipo II (75-90%). La clínica suele comenzar a la semana, con grave afectación del estado general, fontanela abombada y vesículas en la zona de presentación (en la práctica clínica sólo aparecen en un 70% de los casos). Se asocia con una elevada mortalidad. El neonato contrae la infección cuando pasa por el canal del parto, por lo tanto, lesiones genitales activas en el momento del parto contraindican un parto vaginal y es indicación de cesárea.El tratamiento en este niño sería la administración de aciclovir intravenoso.No te olvides de repasar las características de las infecciones connatales.

Pregunta 31.- R: 2El granuloma, que aparece al caerse el cordón (por infección leve o por epitelización incompleta), es un tejido blando, granular, vascular y rojizo o rosado, que puede tener a veces una secreción mucopurulenta. Las masas umbilicales se pueden clasifi car en grandes (hernia umbilical,

en coriorretinitis), manifestación más habitual, aunque suele aparecer de forma tardía calcifi caciones intracraneales difusas, hidrocefalia y convulsiones.Si ocurre en el segundo o tercer trimestre, la infección fetal es más frecuente, pero el RN suele presentar síntomas leves o enfermedad subclínica.

Pregunta 28.- R: 1Estamos ante un niño RN CIR (edad gestacional de 38 semanas y peso < 2.500 g) con clínica de erupción petequial y hepatoesplenomegalia. Esto lo puede presentar cualquier infección connatal.A continuación nos describen la clínica típica del DAP: soplo continuo en 2.º espacio intercostal izquierdo con pulsos femorales saltones. Es un caso de rubéola congénita. Las lesiones óseas también son las típicas de la rubéola, las que nos encontraríamos en la sífi lis serían las mismas, pero con reacción perióstica.

Pregunta 28. Manifestaciones clínicas de la rubéola congénita.De la rubéola congénita para el MIR debemos recordar:

MOMENTO DE ADQUISICIÓN

VÍA DE ADQUISICIÓN

ESTIGMAS CARACTERÍSTICOS

Rubéola Sobre todo 1.er trimestre Placentaria· Tríada de Gregg: catarata, sordera/cardiopatía (ductus sobre todo)· Otros: coriorretinitis, retinopatía sal-pimienta, estenosis pulmonar

periférica, púrpura trombopénica

CMV(la más frecuente)

Más frecuente en 3.er trimestreMás grave en 1.er trimestre: más clínica

Placentaria, canal, leche

· Calcifi caciones periventriculares cerebrales, microcefalia, coriorrenitis· Muchos asintomáticos (con o sin secuelas tardías: la más frecuente es

la sordera)

VHS (75-95% VHS II)

Más frecuente en partoCanal,transplacentario, postparto (90%)

· Precoz: CIR, vesículas, alteraciones neurológicas y oculares (queratoconjuntivitis)

· Tardía (en parto): sepsis, encefalitis con lesión del lóbulo temporal, alteración ocular (sobre todo queratoconjuntivitis y vesículas cutáneas)

ToxoplasmosisMás grave en 1.er trimestre, más frecuente en el 3.er trimestre

Placentaria· Calcifi caciones cerebrales periféricas· Tétrada de Sabin: coriorretinitis, hidrocefalia, convulsiones,

calcifi caciones

VaricelaMás grave en 1.er trimestre. Si sepresenta en el 3.er trimestre, es más grave cuanto más cerca del parto

Placentaria· Precoz: cicatrices, atrofi a de miembros, malformaciones oculares

y cerebrales· Tardía: vesículas, afectación visceral y difi cultad respiratoria

Sífi lis Sobre todo en 3.er trimestre Placentaria

· Precoz (< 2 años): pénfi go sifi lítico, hepatoesplenomegalia, rinitis (tríada de la sífi lis precoz)

· Tardía (> 2 años): tríada de Hutchinson (sordera + queratitis + alteraciones dentarias), articulación de Clutton, periostitis

Pregunta 30. Infecciones connatales.

Page 19: Testcom1v Pd

Comentarios de Test a distancia 1.ª vuelta

Pediatría

CTO Medicina • C/Francisco Silvela, 106 • 28002 - Madrid • Tfno. (0034) 91 782 43 30/33/34 • E-mail: [email protected] • www. ctomedicina. 7

Su densidad es mayor que la leche humana madura (respuesta 3 correcta) pues contiene mayor cantidad de proteínas y minerales que ésta pero no hidratos de carbono (respuesta 2 falsa) ni grasa, y una serie de factores inmunitarios importantes en la defensa del RN (respuesta 5 correcta).Con los días, el calostro es sustituido por una leche de transición que se convierte en madura hacia la 3.ª-4.ª semana (respuesta 4 correcta).

Pregunta 36.- R: 2Es importante recordar las principales características que las dife-rencian.

LECHE HUMANA LECHE DE VACA

Calorías 670 kcal/l 670 kcal/l

Proteínas· 1-1,5 g· Caseína 30%· Seroproteínas 70%

· 3-4,5 g· Caseína 80%· Seroproteínas 20%

Hidratos de carbono 7 g lactosa y otras 4,5 g lactosa

Grasas

· 3,5 g % ác. grasos esenciales

· Ác. grasos cadena larga insaturados

· Colesterol

· 3,5 g· Escasos ác. grasos

esenciales· Ác. grasos

saturados

Minerales + +++ (5 veces)

Hierro + +

Cobre ++ +

Flúor + -

Relación calcio/fósforo 2 1

Vitamina A ++ +

Vitamina B + ++

Vitamina C + Escasa

Vitamina D + Escasa

Vitamina E ++ +

Vitamina K Escasa +

Nitrógeno no proteico

+++ +

Pregunta 36. Comparación entre leche humana y leche de vaca.

Pregunta 37.- R: 3La causa más frecuente de talla baja en el niño son las dos variantes de la normalidad.

TALLA BAJA FAMILIAR

RETRASO CONSTITUCIONAL DEL CRECIMIENTO

Talla RNMenor de la normal

Normal

Velocidad de crecimiento

Normal Menor de lo normal, luego normal

Antecedentes familiares

De talla baja De pubertad retrasada

Edad óseaIgual a edad cronológica

· Igual a edad talla· Retrasada respecto a la edad

cronológica

Talla fi nal Disminuida Normal o algo disminuida

Pregunta 37. Tipos de talla baja no patológica.

onfalocele y gastrosquisis) y pequeñas (granuloma, pólipo y onfalitis). Recuerda las principales características de cada uno de ellos.

Hernia umbilical Onfalocele Gastrosquisis

DEFECTO DE CIERRE Umbilical Umbilical

Paramedio(lat. dcho. + frec.)

CUBIERTAS Peritoneo y piel Peritoneo Sin peritoneo

COMPLICACIONES

- Estrangulación (rara)

- Reducción espontánea

- Anomalías cromosómicas

- Extrofi a vesical- Sd. Beckwith

(macrosomía e hipoglucemia)

Atresia intestinal

TRATAMIENTO

Quirúrgico, si:- Estrangulación- Crecimiento

progresivo- Persiste a los

3-5 años

Corrección quirúrgica precoz(peor pronóstico)

Corrección quirúrgica precoz(buen pronóstico)

Pregunta 31. Patología umbilical.

Sobre secreciones por el ombligo han preguntando en el MIR:• Persistencia del uraco: se segrega por el ombligo un líquido amari-

llento similar a la orina con un pH ácido.• Persistencia del conducto onfalomesentérico (fístula intestinal):

aparece un líquido amarillento con un pH alcalino.

Pregunta 32.- R: 4El hipotiroidismo congénito es una patología relativamente común que se debe descartar en todos los neonatos. Su causa más frecuente es la disgenesia tiroidea (respuesta 1 correcta).En el recién nacido los signos y síntomas de hipotiroidismo son muy sutiles y la clínica se va instaurando de forma progresiva (respuesta 4 falsa). Aparece facies peculiar, estreñimiento, ictericia prolongada, letargia, hernia umbilical, fontanelas amplias, retraso en la madura-ción ósea y puede asociarse una sordera neurosensorial (respuesta 2 correcta). La determinación de TSH en sangre, obtenida entre los 2 y los 5 días de vida, es la prueba de cribado rutinaria. Cuando se confi rma el diagnóstico es necesario iniciar el tratamiento precoz con levotiroxina para evitar la aparición de retraso mental irreversible (respuesta 3 correcta).

Pregunta 33.- R: 4Recordar los siguientes datos de forma muy simplifi cada:• 1,5 m: sonrisa social.• 3 m: inicio sostén cefálico.• 6 m: inicia sedestación.• 9 m: movimiento de pinza.• 10-12 m. Inicia bipedestación.• 12-15 m: primeros pasos y emite bisílabos.• 24 m: sube/baja escaleras, corre y apila 4 o 6 cubos para formar una

torre.• 5-10 años: comprende que la muerte es un fenómeno permanente.

Pregunta 34.- R: 5Recuerda que el peso del niño se duplica al 5.º mes, se triplica al año y se cuadriplica a los 2 años. La talla del nacimiento se duplica alrededor de los 4 años de edad (respuesta 5 falsa).

Pregunta 35.- R: 2Del calostro debes saber que es la leche de los 2-4 primeros días después del parto (respuesta 1 correcta).

Page 20: Testcom1v Pd

Comentarios de Test a distancia 1.ª vuelta

CTO Medicina • C/Francisco Silvela, 106 • 28002 - Madrid • Tfno. (0034) 91 782 43 30/33/34 • E-mail: [email protected] • www. ctomedicina.com 8

Pediatríaseguridad (UCI, quirófano) y administración de antibióticos, general-mente cefalosporinas de 3.ª generación. Los corticoides pueden ser benefi ciosos en la primera parte del tratamiento pero la adrenalina no es útil (respuesta 3 falsa). Con el tratamiento adecuado, el cuadro suele remitir en 24-48 horas.En los casos dudosos nunca hay que intentar visualizar la epiglotis, sino realizar una Rx lateral de faringe donde se apreciará una epiglotis engrosada.

Pregunta 42.- R: 5Estamos ante un cuadro de bronquiolitis aguda que se defi ne como primer episodio de difi cultad respiratoria con sibilancias. El agente más frecuente es el VRS. Es una enfermedad estacional de los meses de in-vierno y primavera.Afecta a niños menores de 2 años, principalmente lactantes. Comienza por un cuadro catarral, con tos, mocos y febrícula, y en los días siguientes evolu-ciona hacia un cuadro de obstrucción de vías aéreas distales con sibilancias (dato más típico de la exploración), espiración alargada, roncus dispersos e hipoventilación. Esta fase crítica suele durar 2-3 días con posterior mejoría (respuestas 1 y 3 correctas). En lactante muy pequeño pueden producirse pau-sas de apnea con cianosis como única manifestación (respuesta 2 correcta).El tratamiento consiste en medidas generales (oxigenoterapia, monito-rización, etc.), beta-2 agonistas inhalados y adrenalina nebulizada. Ni los corticoides ni los antibióticos están indicados.La ribavirina inhalada estaría recomendada en bronquiolitis graves que afectan a niños de riesgo (patología pulmonar o cardíaca de base). Actualmente se intenta prevenir su aparición con la administración de palivizumab, anticuerpo monoclonal anti-VRS indicado en prematuros, cardiópatas, neumópatas e inmunodeprimidos.La secuela a largo plazo más frecuente es la hiperreactividad bronquial. La bronconeumonía y OMA son poco frecuentes (respuesta 5 falsa).

Pregunta 43.- R: 4El cuadro que presenta este niño es compatible con una fi brosis quística.Las manifestaciones clínicas suelen ser:• Respiratorias: bronquiolitis recurrentes, tos crónica, neumonía, pólipos

nasales, sinusitis, etc.• Digestivas: íleo meconial, esteatorrea, malabsorción, prolapso rectal,

cirrosis biliar, etc.• Otras: deshidratación, malnutrición, diabetes, azoospermia.

Pregunta 38.- R: 5Si tenemos presente la tabla del comentario de la pregunta anterior podemos deducir fácilmente que estamos ante un caso de talla baja familiar. La curva de crecimiento es inferior al percentil 3, no hay dis-cordancia entre la edad ósea del niño y su edad cronológica y, además, existe historia familiar de talla baja. Las pruebas de laboratorio serán normales y la talla adulta fi nal previsible será baja, pero dentro de los límites esperados para su talla genética.

Pregunta 39.- R: 3Es un cuadro de laringitis aguda, obstrucción de la vía aérea superior, de etiología viral (virus parainfl uenzae en el 75% de los casos) caracterizado por un antecedente previo de infección de vías altas con fi ebre de 38-38,5 ºC, tos perruna o metálica, afonía, estridor de predominio inspiratorio y difi cultad respiratoria variable con empeoramiento nocturno.Afecta a niños entre 3 meses y 5 años y tiene predominio estacional, produciéndose la mayoría de las veces en otoño.El tratamiento consiste en humedad ambiental, corticoides (en aerosol o sistémicos) y adrenalina en aerosol.

Pregunta 40.- R: 2Es un cuadro clínico típico de laringitis pero sin fi ebre y sin ningún an-tecedente de infección de vías altas de características virales. Se trata, por tanto, de una laringitis espasmódica o estridulosa. Es el momento de repasar las características de los tipos de crup.

Pregunta 41.- R: 3La epiglotitis es una enfermedad típica de los niños menores de 5 años. En la actualidad, los principales responsables de esta entidad son los cocos grampositivos como el S. pyogenes, S. pneumoniae y S. aureus. Por detrás de éstos, hay que tener en cuenta también a su agente clásico, H. infl uenzae tipo B (cada vez menos frecuente gracias a la vacunación universal).La clínica suele cursar de forma brusca por la noche, con fi ebre alta y aspecto séptico, babeo e intensa difi cultad respiratoria con estridor inspiratorio. Empeora con el llanto y con el decúbito supino, haciendo que el niño permanezca en “posición de trípode”. No suele existir tos ni afonía y el estridor no es tan ruidoso como el de las laringitis.En el tratamiento, lo prioritario es asegurarse la vía aérea mediante intubación (a ser posible nasotraqueal), siempre en condiciones de

LARINGITIS AGUDA(estridulosa)

LARINGOTRAQUEITIS VIRAL EPIGLOTITIS AGUDATRAQUEÍTIS

BACTERIANA

Etiología Alergia + psicológicoVirus parainfl uenzae 1 (el más frecuente)

Cocos grampositivos S. aureus

Antecedentes No hayCatarro vías altas (paciente y/o familiar)

- Crup vírico

Clínica Espasmo laríngeo recortado (generalmente nocturno) No fi ebre

Fiebre, tos, disnea alta, estridor inspiratorio

Fiebre alta + babeo + disfagia + disnea + cabeza extendida

Fiebre + estridor mixto

Duración 1-2 noches Días-semanas Fulminante Días-semanas

Tratamiento· Ambiente tranquilo, humidifi car· Corticoides· Adrenalina racémica en aerosol

· Intubar + oxígeno· Antibiótico· Tto. en UVI

· Oxígeno· Intubación si es

preciso· Cloxacilina i.v.

Pregunta 40. Tipos de crup.

Page 21: Testcom1v Pd

Comentarios de Test a distancia 1.ª vuelta

Pediatría

CTO Medicina • C/Francisco Silvela, 106 • 28002 - Madrid • Tfno. (0034) 91 782 43 30/33/34 • E-mail: [email protected] • www. ctomedicina. 9

Pregunta 48.- R: 1Estamos ante un caso de alergia a proteínas de leche de vaca. El cuadro típico consiste en inicio de la clínica tras la ingesta de un biberón de fórmula adaptada de leche de vaca. La proteína más frecuentemente implicada es la betalactoglobulina (respuesta 2 correcta). La sintoma-tología más habitual es la cutánea (urticaria), seguida de la digestiva (vómitos, diarrea) y en ocasiones respiratoria (bronco espasmo). El cuadro está mediado por un mecanismo inmunológico tipo IgE. Afecta a individuos atópicos o con antecedentes familiares de atopia (respuesta 1 falsa). El diagnóstico es fundamentalmente clínico, se basa en la exclusión de dichas proteínas y consecuente desaparición de los síntomas y la posterior provocación (respuesta 4 correcta). Se emplean las pruebas cutáneas, que serán positivas (respuesta 3 correcta). El tratamiento consiste en la suspensión de la fórmula adaptada, reem-plazándola por fórmulas especiales (hidrolizado de proteínas). Suele ser un proceso autolimitado (respuesta 5 correcta), la mayoría de los niños pueden tomar leche de vaca a partir de los 2 años.Recuerda las diferencias entre INTOLERANCIA y ALERGIA a las proteínas de leche de vaca.

APLV IPLV

Patogenia Mediado IgE No IgE

Síntomas Digestivos y alérgicos Digestivos

Diagnóstico

ClínicoProvocación

· InmunoCAP · Prick

Tratamiento

HIDROLIZADOS DE PLV

· AntiH1· Adrenalina s.c.

Pregunta 48. Intolerancia-alergia a proteínas de leche de vaca.

Pregunta 49.- R: 4Caso clínico de atresia con fístula traqueoesofágica. Según la clasifi -cación de Ladd, el tipo más frecuente es la atresia proximal con fístula distal (tipo III).El diagnóstico se realiza por la sospecha clínica (salivación excesiva, cianosis y atragantamiento con las tomas, etc.) e imposibilidad para pasar una sonda nasogástrica. El tratamiento es quirúrgico, siendo el refl ujo gastroesofágico la com-plicación postquirúrgica más frecuente, que suele ser grave (respuesta 4 correcta). Otras complicaciones menos habituales son la fístula de la anastomosis, la recidiva de la fístula traqueoesofágica, la estenosis esofágica y la traqueomalacia.

Pregunta 50.- R: 5Estamos ante un caso de RGE fi siológico, caracterizado por pequeños vómitos sin fuerza después de las tomas, por lo demás, el niño se en-cuentra asintomático con peso y talla en percentiles normales.El RGE es muy frecuente en el primer año de vida, hasta un 85%. Gene-ralmente desaparece hacia los 2 años.En un 10% de los casos, se habla de RGE patológico o enfermedad por refl ujo gastroesofágico (ERGE) cuando se asocia con escasa ganancia ponderal, esofagitis y clínica respiratoria.El diagnóstico se basa en la historia clínica. Sólo en el RGE patológico está indicada la realización de pruebas complementarias: tránsito superior con bario (primera prueba a realizar) y una pHmetría de 24 horas (para cuantifi car el grado de refl ujo es la prueba más sensible y específi ca, se

Para el diagnóstico se requiere manifestaciones clínicas compatibles y/o antecedente de hermano/a con enfermedad confi rmada y/o cribado neonatal positivo más un criterio de confi rmación (test del sudor positivo o presencia de mutaciones en el estudio genético o test de diferencia de potencial nasal positivo). Para el tratamiento de estos pacientes es fundamental el cuidado del pulmón. Se debe realizar diariamente fi sioterapia respiratoria y trata-miento agresivo de las infecciones pulmonares. Además, se llevará a cabo un adecuado soporte nutricional y tratamiento específi co de las posibles complicaciones.

Pregunta 44.- R: 1La traqueítis bacteriana tiene como principal responsable a S. aureus (respuesta 1 falsa). Hay que sospechar esta entidad cuando, tras un cuadro de crup vírico, se produce un empeoramiento progresivo con fi ebre, difi cultad respiratoria de intensidad creciente y estridor mixto.Es una entidad grave que suele requerir la hospitalización del niño para instaurar tratamiento antibiótico (cloxacilina i.v.) y oxigenoterapia a demanda. Si aparece gran difi cultad respiratoria puede ser necesaria la intubación.

Pregunta 45.- R: 4En la patogenia de la fi brosis quística está implicada una alteración de la regulación de los canales iónicos de las membranas celulares, lo que produce secreciones deshidratadas y espesas. A nivel pulmonar, la le-sión anatomopatológica inicial es la bronquiolitis (respuesta 1 correcta).El íleo meconial supone la forma de debut neonatal en aproximadamente un 10% de los casos (respuesta 2 correcta). La función endocrina del páncreas se afecta con el tiempo, pudiendo aparecer diabetes mellitus a partir de los 10 años de evolución de la enfermedad (respuesta 3 correcta).Debido a la pérdida excesiva de sal, se ven cuadros de deshidratación con hiponatremia e hipocloremia, coincidiendo con gastroenteritis o en épocas de calor (respuesta 5 correcta).La aspergilosis broncopulmonar alérgica es relativamente frecuente en estos pacientes. Hay que sospecharla ante la presencia de esputo herrum-broso, aislamiento de Aspergillus fumigatus o la presencia de eosinófi los en una muestra fresca de esputo. El tratamiento será corticoterapia oral, en los casos refractarios puede ser necesario el empleo de anfotericina B en aerosol o de 5-fl uorocitosina sistémica (respuesta 4 falsa).

Pregunta 46.- R: 4Es necesario iniciar el tratamiento de la fi brosis quística antes de que la afectación pulmonar sea importante. Se ha visto que el empleo de fi sioterapia respiratoria alarga la supervivencia de estos pacientes. Es útil el empleo moderado de enzimas pancreáticas para disminuir la esteatorrea, así como el aporte de suplementos de vitaminas liposolu-bles a la dieta.En los pacientes con fi brosis quística, la neumonía por sobreinfección bacteriana se debe principalmente a Pseudomonas aeruginosa, variedad mucoide. Este patógeno puede colonizar la vía respiratoria dando lugar a un estado de portador crónico difícil de revertir (respuesta 4 falsa).

Pregunta 47.- R: 4La causa más frecuente de GEA en nuestro medio es el rotavirus. Es una enfermedad típica de lactantes que produce una diarrea líquida, SIN sangre. Es proceso autolimitado, que cede en 3-10 días (respuesta 4 falsa). El diagnóstico se basa en la realización de un test de ELISA en las heces para detectar el Ag.El tratamiento es sintomático. Existen dos tipos vacunas atenuadas de 2 o 3 dosis respectivamente de reciente aparición y administra-ción oral.

Page 22: Testcom1v Pd

Comentarios de Test a distancia 1.ª vuelta

CTO Medicina • C/Francisco Silvela, 106 • 28002 - Madrid • Tfno. (0034) 91 782 43 30/33/34 • E-mail: [email protected] • www. ctomedicina.com 10

Pediatría

lactantes (respuesta 1 correcta). Se localiza a unos 50-70 cm de la válvula ileocecal (respuesta 2 correcta).La clínica más frecuente suele consistir en una hemorragia rectal IN-DOLORA (respuesta 4 falsa) e intermitente debido a la ulceración de la mucosa ileal adyacente al divertículo con mucosa ectópica (gástrica o pancreática).La técnica diagnóstica más sensible es la gammagrafía con Tc-99.El tratamiento es quirúrgico.

Pregunta 54.- R: 3Cuadro típico de invaginación intestinal. Recuerda que es la causa más frecuente de obstrucción intestinal entre los 3 meses y los 6 años. Niño que presenta episodios repetidos de llanto y encogimiento de piernas con posterior decaimiento. Si la clínica evoluciona hasta un 60%, puede presentar deposiciones “en jalea de grosella”, heces con sangre roja fresca y moco. La forma más frecuente es la ileocólica.Para el diagnóstico inicial se suele realizar una ecografía abdominal.Una invaginación intestinal es siempre una urgencia. Si han transcurrido menos de 48 horas y no hay signos de perforación, se realiza tratamien-to con enemas de bario o con aire, para intentar la reducción. Si hay perforación intestinal o distensión abdominal de más de 48 horas de evolución, se prefi ere tratamiento quirúrgico.La tasa de recurrencia es mayor en los pacientes en los que se realiza reducción hidrostática (10-20%) que en los tratados quirúrgicamente (3%).

Pregunta 55.- R: 4La enfermedad de Hirschprung o megacolon congénito se caracteriza por estreñimiento crónico desde el nacimiento, asociado a un estanca-miento o retraso ponderal (por malabsorción) secundario a un trastorno en la migración de los neuroblastos que supone la ausencia de células ganglionares y plexos mesentéricos.Para el diagnóstico es útil el tacto rectal, donde se aprecia una am-polla rectal vacía de heces (respuesta 3 correcta). En la manometría se observa un aumento paradójico del tono del esfínter anal interno (respuesta 4 falsa) ante un incremento de presión a ese nivel. En el enema opaco se objetiva un retraso en la eliminación de contraste (respuesta 5 correcta). El diagnóstico defi nitivo lo dará la biopsia: en el segmento afectado se observa ausencia de células ganglionares con incremento de la acetilcolinesterasa y aumento de las terminaciones nerviosas (respuesta 1 correcta).El tratamiento es quirúrgico, con la resección de todo el segmento agangliónico. En el RN con obstrucción intestinal se realiza colostomía y descenso con posterior cierre de colostomía. Si no hay obstrucción, se realiza un descenso primario precoz.

realiza cuando existen manifestaciones extradigestivas del refl ujo). El tratamiento en los casos leves consiste en medidas posturales y espe-santes de las tomas. En casos de refl ujos más graves se administraran estimulantes del peristaltismo (cisaprida, domperidona) o tratamiento quirúrgico (funduplicatura de Nissen).

Pregunta 51.- R: 5La clínica típica de un cuadro de estenosis hipertrófi ca de píloro consiste en vómitos proyectivos inmediatos tras las tomas, alimenticios, no biliosos, tras el vómito el niño se queda irritable y hambriento. Comienza entre las 3-6 semanas de vida, siendo el momento más habitual en torno a los 20 días de vida (respuesta 1 falsa). Es más habitual en varones de raza blanca (respuesta 2 falsa).Debido a las pérdidas de hidrogeniones y cloruros con los vómitos se produce una alcalosis metabólica hipoclorémica (respuesta 3 falsa) con normopotasemia o hipopotasemia.La prueba diagnóstica de elección ante la sospecha de este cuadro sería una ecografía abdominal (respuesta 4 falsa). El hallazgo frecuente en la radiografía simple de abdomen es la distensión gástrica con escaso gas distal. El tránsito digestivo está en desuso en la actualidad, se reserva para pacientes en los que la ecografía no es concluyente, donde aparece de forma típica la imagen del “signo de la cuerda”.El tratamiento es quirúrgico: pilorotomía extramucosa de Ramsted (res-puesta 5 correcta). Recuerda que pueden existir vómitos postoperatorios secundarios al edema de píloro producidos por la propia incisión. Repasa los cuadros clínicos que producen vómitos en el lactante.

Pregunta 52.- R: 4Caso típico de atresia duodenal con vómitos biliosos en las primeras 24- 48 horas, tras las primeras tomas, con abdomen excavado por ausencia de aire distal. Importante el antecedente de síndrome de Down, más común en estos niños. Con mayor frecuencia la atresia se localiza a nivel de la 3.ª porción del duodeno. En la Rx de abdomen es típico encontrar la imagen de “doble burbuja”. Recuerda los datos clínicos característicos: • Estenosis hipertrófi ca de píloro: vómitos no biliosos a las 2-3 semanas

de vida.• Enfermedad de Hirschprung: estreñimiento y en ocasiones vómitos

fecaloideos.• Divertículo de Meckel: sangrado indoloro rectal sin vómitos.• Invaginación intestinal: episodios de dolor intenso, siendo muy raro

en el periodo neonatal.

Pregunta 53.- R: 4El divertículo de Meckel es un resto del conducto onfalomesentérico. Es la malformación digestiva más frecuente, apareciendo en un 2% de

REFLUJO GASTROESOFÁGICOESTENOSIS HIPERTRÓFICA

DE PÍLOROATRESIA PILÓRICA ATRESIA DUODENAL

Momento de aparición 1.ª semana 2.ª a 3.ª semana

Vómitos Alimentarios sin fuerza Alimentarios a chorro Alimentarios Biliosos

Otros síntomas• Síntomas respiratorios• Disminución del crecimiento

• Masa palpable• Alcalosis hipoclorémica

• Abdomen excavado• Asociar al Down

Diagnóstico

Clínica + respuesta a tratamiento

pHmetría 24 h

endoscopia

Evolución• Rx: “signo de la cuerda”• 1 burbuja gástrica

RX: • 1 burbuja gástrica• Ausencia de gas distal

RX: • 1 burbuja gástrica• 1 burbuja duodenal

Tratamiento• Medidas posturales• Cisaprida• Cirugía (funduplicatura de Nissen)

Cirugía: • Piloromiotomía• Fredet-Ramsted

Cirugía Cirugía

Pregunta 51. Vómitos del lactante.

Page 23: Testcom1v Pd

Comentarios de Test a distancia 1.ª vuelta

Pediatría

CTO Medicina • C/Francisco Silvela, 106 • 28002 - Madrid • Tfno. (0034) 91 782 43 30/33/34 • E-mail: [email protected] • www. ctomedicina. 11

Se hará, al menos en una ocasión, estando el paciente consumiendo gluten (respuesta 4 falsa). Una vez asegurado el diagnóstico se iniciará el tratamiento a base de una dieta rigurosamente exenta de gluten de por vida.La biopsia intestinal en el caso de la enfermedad celíaca no es patog-nomónica (respuesta 3 falsa), pero sí muy característica: aparece un infi ltrado infl amatorio en la lámina propia, junto con hiperplasia de las criptas y atrofi a vellositaria.El seguimiento se realizará valorando la evolución clínica y determinando periódicamente los anticuerpos para evaluar el cumplimiento de la dieta (respuesta 5 falsa). Los anticuerpos más sensibles y específi cos son los IgA antitransglutaminasa (respuesta 2 correcta).

Pregunta 58.- R: 2Estamos ante un cuadro de atresia de vías biliares extrahepáticas (AVBEH); lo típico es RN a término sano y con fenotipo normal que desarrolla cuadro de colestasis (predominio de bilirrubina directa, coluria, acolia o hipocolia) a las 2-3 semanas de vida. Se puede asociar a poliesplenia, malrotación intestinal, anomalías vasculares, etc. La aparición de varios casos en la misma familia es rara.La prueba diagnóstica sería la realización de un HIDA, donde se aprecia captación del contraste por el hígado, pero que no llega al intestino (au-sencia de excreción). El diagnóstico de certeza se consigue mediante la-parotomía exploradora con realización intraoperatoria de colangiografía.Su tratamiento defi nitivo es el trasplante hepático (el 80% de pacientes lo requerirá a lo largo de su evolución). Previamente se intentará realizar una hepatoportoenterostomía o técnica de Kasai, para intentar restable-cer el fl ujo biliar y disminuir el daño (respuesta 2 falsa). Actualmente, la AVBEH constituye la primera indicación del trasplante hepático infantil.

Pregunta 58. Colestasis neonatal.

Pregunta 59.- R: 3Es un cuadro de malabsorción de hidratos de carbono. Tiene como an-tecedente una GEA que lesiona el borde del enterocito, produciendo un défi cit transitorio de lactasa. La clínica se caracteriza por diarrea, dolor cólico, heces espumosas y ácidas (pH < 5,5) que excorian la zona del pañal. Ante la sospecha se debe realizar un Clinitest en las heces que será positivo. El tratamiento consiste en la exclusión de la lactosa de la dieta, que la mayoría de las veces es transitoria.

Pregunta 56.- R: 5Los factores que intervienen en la patogenia de la celiaquía son:• HLA: DR3, DR4, DR7 , DQ y W2.• Factores inmunológicos: los linfocitos de la lámina propia se inmu-

nizan frente a la gliadina.• Factores ambientales.

La clínica se caracteriza por un cuadro de malabsorción, de inicio más frecuente entre los 6 meses y los 2 años. Suelen presentar esteatorrea, estancamiento de la curva ponderal, disminución de la masa muscular, irritabilidad y distensión abdominal importante.El diagnóstico se basa en los test serológicos, Acs antigliadina, antirre-ticulina, antiendomisio y antitransglutaminasa, estos últimos los más sensibles y específi cos de todos, y en la realización de biopsia intestinal, necesaria para establecer diagnóstico. La biopsia se hará, al menos, en una ocasión, estando el paciente consumiendo gluten y se tomarán varias muestras de porción duodenal o inicio de yeyuno. Clásicamente se realizaban tres biopsias: 1.ª (sin exclusión), demuestra atrofi a; 2.ª (tras exclusión), demuestra recuperación de mucosa; y 3.ª (tras prueba de provocación), demuestra atrofi a. Actualmente la 2.ª y la 3.ª se realizan únicamente cuando:• El inicio de la clínica se produjo por debajo de los 2 años de edad

(riesgo de falsos positivos en la biopsia).• La respuesta clínica a la exclusión del gluten de la dieta no ha sido

concluyente y existe duda diagnóstica.• La retirada del gluten se hizo sin realización de primera biopsia.

Para llevar a cabo la 2.ª biopsia, se debe esperar un mínimo de 24 meses tras retirada del gluten y no ha de practicarse hasta los 6 años de vida.El tratamiento consiste en la exclusión de por vida del trigo, cebada, cen-teno y +/- avena y triticale. Pueden tomar libremente maíz, soja y arroz.

Pregunta 56. Diagnóstico de la enfermedad celíaca.

Pregunta 57.- R: 2Pregunta importante sobre el diagnóstico de la enfermedad celíaca. Ante la presencia de una alta sospecha clínica es necesaria la realización de una biopsia intestinal para confi rmar el diagnóstico (respuesta 1 falsa).

Page 24: Testcom1v Pd

Comentarios de Test a distancia 1.ª vuelta

CTO Medicina • C/Francisco Silvela, 106 • 28002 - Madrid • Tfno. (0034) 91 782 43 30/33/34 • E-mail: [email protected] • www. ctomedicina.com 12

PediatríaPregunta 63.- R: 4Caso clínico típico de síndrome hemolítico urémico (SHU); niño menor de 4 años con antecedente de gastroenteritis enteroinvasiva que presenta signos y síntomas de insufi ciencia renal aguda, clínica neurológica y afectación gastrointestinal. El agente más frecuente es E. coli O-156). El patógeno produce unas toxinas que favorecen la lesión a nivel endotelial capilar y arteriolar con la formación de trombos de plaquetas con trombopenia de consumo y anemia microangiopática (respuesta 2 correcta). El órgano más afectado en este cuadro es el riñón y es la causa más frecuente de IRA en menores de 5 años (res-puesta 1 correcta).El diagnóstico se basa en:• Anemia hemolítica microangiopática: haptoglobina descendida con

esquistocitos en el frotis de sangre periférica.• Plaquetopenia.• IRA: por ecografía se debe diferenciar de la trombosis venosa bilateral

(respuesta 3 correcta).

Se realizará tratamiento conservador de la IRA y alteraciones hidro-electrolíticas y si no consiguen controlarse, estará indicada la diálisis peritoneal (respuesta 4 falsa).La función renal se recupera en el 90% de pacientes y las recidivas son poco frecuentes.

Pregunta 64.- R: 2Estamos ante un cuadro de torsión testicular. Se debe realizar diagnóstico diferencial con la orquioepididimitis.La clínica consiste en un dolor agudo intenso, con tumefacción escrotal, sin fi ebre ni traumatismo previo. El refl ejo cremastérico suele estar anulado.El diagnóstico se basa fundamentalmente en la clínica, en caso de duda se puede recurrir a la gammagrafía de fl ujo testicular o a la realización de una eco-Doppler.El tratamiento consiste en la reducción manual o fi jación quirúrgica del teste afecto y del contralateral. El tiempo de viabilidad del testículo torsionado es de 4-6 horas, por lo que siempre se debe considerar una urgencia quirúrgica.Recuerda las causas de escroto agudo.

Dolor Estado generalRefl ejo

cremastéricoInfl amación

TORSIÓNTESTICULAR

· Intenso· Brusco· Continuo

· Afectación importante

· Cortejo vegetativo

· Abolido· Teste

ascendidoo transverso

· Teste induradoy edematoso

· Eritema escrotal

TORSIÓNDEL

HIDÁTIDE

· Menos intenso

· Gradual· Dolor a la

palpación del polo superior

Mínima afectación

Presente · Menos intensa

· Punto azulen polo superior

ORQUIOEPI-DIDIMITIS

· Menos intenso

· Progresivo

Síndromemiccional asociado

Presente · Moderada· Edema

escrotal

Pregunta 64. Diagnóstico diferencial del escroto agudo.

Pregunta 65.- R: 3Caso clínico de PTI (púrpura trombótica idiopática). Típicamente apa-rece como un cuadro de petequias generalizadas, con disminución del número de plaquetas, tras un catarro de vías altas. No suele existir

• El défi cit de sacarasa-isomaltasa: produce la misma clínica pero el Clinitest es negativo.

• El défi cit aislado de isomaltasa: es asintomático.• El défi cit de enteroquinasa: produce una malabsorción de proteínas

con clínica de desnutrición y edemas.• Gastroenteritis por rotavirus: la clínica es vómitos y diarrea líquida,

no ácida.

Pregunta 60.- R: 2La criptorquidia (descenso testicular incompleto) es el trastorno de la diferenciación sexual más frecuente en los varones. Se estima que un 4,5 % de los niños tienen criptorquidia al nacimiento. Como el descenso testicular se produce en la última fase de la gestación, es más frecuente en prematuros. La mayoría de los testículos no descendidos bajan al escroto espontáneamente en los primeros 3 meses de vida. Si ésto no ha ocurrido a los 6 meses de edad, es poco probable que lo haga más adelante. Las consecuencias de la criptorquidia permanente son:• Riesgo de esterilidad en los casos bilaterales.• Riesgo de degeneración maligna a seminoma.• Hernias asociadas.• Torsión testicular en el lado afecto.• Efectos psicológicos.

La edad idónea para realizar el tratamiento es entre los 9 y los 15 meses de edad, preferentemente antes de los 2 años. Consiste en el descenso y la fi jación quirúrgica del testículo al escroto (orquidopexia); en todo caso es primordial realizarla antes de la pubertad, sobre todo para eliminar el riesgo de degeneración maligna. El tratamiento con HCG es más efi caz cuanto mayor es la edad del niño y con una localización baja del teste.

Pregunta 61.- R: 4En un lactante que presenta un cuadro clínico inespecífi co (vómitos, febrícula y pérdida de apetito), junto con leucocitosis, leucocituria y nitritos positivos en orina, debemos sospechar una infección del tracto urinario. Recuerda que la tira reactiva de orina es un método sensible que permite seleccionar aquellos niños a los que hay que realizar un cultivo de orina, en este caso obtenido mediante punción suprapúbica, e iniciar el tratamiento antibiótico.

Pregunta 62.- R: 3Respecto al RVU, hay que saber que es la anomalía congénita más fre-cuente de la unión ureterovesical. Se suele diagnosticar al estudiar las ITU porque es causa predisponente de la misma. El paso retrógrado de orina desde la vejiga al uréter y pelvis renal desencadena una reacción infl amatoria que puede dar lugar a la formación de cicatrices renales y si éstas son extensas, se afectará la función renal (nefropatía por refl ujo, que es la causa de hasta un 20% de las insufi ciencias renales y la primera causa de HTA en la infancia).Para el diagnóstico, la técnica de elección es la cistografía miccional que permite clasifi car el refl ujo según intensidad, grado de dilatación ureteral y deformidad calicial (grados I-V). Se debe realizar también una ecografía renal para descartar anomalías estructurales y una gammagrafía renal, técnica de referencia para el diagnóstico de cicatrices renales.El refl ujo grado I y II en el 80% de los casos desaparece de forma espon-tánea al madurar el niño y no precisan tratamiento. Los objetivos del tratamiento son prevenir la pielonefritis y la lesión renal. • La profi laxis antibiótica a dosis básica es el pilar fundamental del

refl ujo leve hasta que éste desaparezca.• En los casos que por su grado y repercusiones morfológicas a nivel

renal, se espera que no desaparezca y/o lesione más el riñón, se obtará por la cirugía, que puede ser endoscópica o abierta.

Page 25: Testcom1v Pd

Comentarios de Test a distancia 1.ª vuelta

Pediatría

CTO Medicina • C/Francisco Silvela, 106 • 28002 - Madrid • Tfno. (0034) 91 782 43 30/33/34 • E-mail: [email protected] • www. ctomedicina. 13

el año y los 5 años. Se han encontrado deleciones en el cromosoma 11 (respuesta 1 correcta). La clínica más frecuente es la masa abdominal asintomática (respuesta 5 correcta) que no suele pasar línea media. Otros síntomas son la HTA, en el 60% de pacientes por compresión arterial por el tumor y producción de renina (respuesta 3 correcta), hematuria, etc. La localización más común de las metástasis es el pulmón. El diagnóstico se basa en la TC y la PAAF, no aconsejándose la realización de biopsia (respuesta 4 falsa).

Pregunta 69.- R: 5Ante un niño que debuta de forma brusca con un síndrome constitu-cional junto con pancitopenia y hepatoesplenomegalia, el diagnóstico más probable será el de leucemia aguda (respuesta 5).

Pregunta 70.- R: 4El tumor que tiene tendencia a presentar calcifi caciones intraneoplásicas (hasta en un 80% de los casos) es el neuroblastoma y no el tumor de Wilms (respuesta 4 falsa).Con las pruebas de imagen se puede confi rmar que el tumor de Wilms (nefroblastoma) es de origen intrarrenal (respuesta 2 correcta). En oca-siones, los pacientes con nefroblastoma pueden presentar policitemia debido a la producción de eritropoyetina (respuesta 1 correcta).Las metástasis pulmonares son las más frecuentes dentro del tumor de Wilms, pero su presencia no suele contraindicar la cirugía del tumor (respuesta 3 correcta).El tumor de Wilms puede ser bilateral, sobre todo en las formas familiares (5%), (respuesta 5 correcta).

Pregunta 71.- R: 4Estamos ante un cuadro de sarampión. El caso clínico va relatando detalladamente las fases de esta enfermedad: • Fase prodrómica: catarro con tos y fi ebre, manchas de Koplik (lesiones

blanquecinas sobre base eritematosa en mucosa subyugal), es un dato patognomónico de esta enfermedad.

• Fase exantemática: inicio del exantema por la cara y descenso (res-puesta 1 correcta). El exantema es maculopapular, no pruriginoso (respuesta 4 falsa) y la gravedad de la enfermedad se relaciona con la intensidad y confl uencia del exantema (respuesta 3 correcta). Es en esta fase cuando aparece fi ebre alta.

• Fase de resolución: el exantema desaparece en el mismo orden que apareció.

Entre las complicaciones fi guran:• Cuadros ORL (otitis media aguda): complicación más frecuente.• Neumonías: en los niños es más común la sobreinfección bacteriana,

que la neumonía de células gigantes de Hecht (respuesta 2 correcta).• Afectación del SNC: encefalitis aguda y PEES (panencefalitis es-

clerosante subaguda): forma de encefalitis por virus lentos, con un pronóstico pésimo, el diagnóstico se hace detectando en LCR aumento de Acs frente al sarampión.

• Anergia cutánea con reactivación de una tuberculosis preexistente (respuesta 5 correcta).

Pregunta 72.- R: 3Con los datos de febrícula y adenopatías retroauriculares y cervicales dolorosas debemos pensar en una rubéola. Se precede de un cuadro catarral leve (1 o 2 días) con fi ebre baja o moderada y conjuntivitis. El exantema es morbiliforme y confl uente en cara. Se resuelve mediante una mínima descamación. En la analítica se puede encontrar leucopenia, trombopenia y linfocitos atípicos.

anemia ni otros signos de diátesis hemorrágica. El pronóstico es bueno, evolucionando hacia la recuperación espontánea en la mayoría de los casos (respuesta 3 cierta).

Pregunta 66.- R: 4Es importante que recuerdes:• Cáncer infantil más frecuente: leucemias (LAL tipo B). Los linfomas

siguen a las leucemias en frecuencia, salvando el grupo de edad de entre 10 y 14 años, en que superan a las anteriores.

• Cáncer sólido infantil más frecuente: tumores del SNC (astrocitoma).• Tumor sólido extracraneal más frecuente: neuroblastoma. El neuro-

blastoma, es más habitual en varones y en la raza blanca, y el 90% se diagnostican por debajo de los 5 años de edad.

• Tumor abdominal más frecuente: neuroblastoma.• Masa abdominal más habitual en el RN: hidronefrosis.• En cuanto a los tumores óseos, en niños mayores y adolescentes,

predomina el osteosarcoma, seguido del sarcoma Ewing, que es el más frecuente en menores de10 años.

Pregunta 67.- R: 3El cuadro que se expone es una niña con un neuroblastoma. La edad de aparición típica es en menores de 2 años. Se suele localizar en el abdo-men a nivel de las glándulas suprarrenales y la clínica más frecuente es la masa abdominal que cruza línea media. Se asocia a hematoma palpebral y hepatomegalia y entre los síndromes paraneoplásicos (no modifi can el pronóstico) fi gura el opsoclonus-mioclonus (caso clínico) y la diarrea secretora. Para el diagnóstico se utiliza la TC craneal, catecolaminas en orina aumentadas y la gammagrafía con MIBG. El tratamiento depende del estadio, realizándose cirugía, quimioterapia y radioterapia. La supervivencia es del 50%. Recuerda las principales diferencias entre neuroblastoma y nefroblastoma.

NEUROBLASTOMA WILMS (NEFROBLASTOMA)

Epidemiología

· Tumor sólido extracraneal más frecuente en niños

· Sobre todo < 2 años· Deleción cromosoma 1

· 2.º tumor abdominal más frecuente en niños

· Mayores que los niños del neuroblastoma (≥ 3 años)

· Deleción cromosoma 11

Localización

· 70% abdomen (suprarrenal o paramedial)

· Pasa línea media· 20% tórax

· Riñón· No pasa línea media· A veces bilateral (familiares)

Clínica

· Masa · Sd. paraneoplásicos

(VIP, opsoclono-mioclono)

· Asocia a veces hemihipertrofi a, aniridia(sd. de Beckwith-Wiedemann) y malformaciones genitourinarias

· Masa abdominal· HTA

Tratamiento· Cirugía en estadio I· Otros: Qx + QT + RT

· Bueno. A veces se diferenciao regresa espontáneamente

· Metástasis a hígado, m.o., piel y hueso

Pronóstico· Qx· RT· QT (según estadio)

· Bueno, sobre todo en menores de 2 años

· Metástasis a pulmón

Pregunta 67. Neuroblastoma vs. nefroblastoma.

Pregunta 68.- R: 4El tumor de Wilms se asocia con anomalías genitourinarias, hemihipertro-fi a y aniridia (respuesta 2 correcta). La edad de diagnóstico oscila entre

Page 26: Testcom1v Pd

Comentarios de Test a distancia 1.ª vuelta

CTO Medicina • C/Francisco Silvela, 106 • 28002 - Madrid • Tfno. (0034) 91 782 43 30/33/34 • E-mail: [email protected] • www. ctomedicina.com 14

Pediatría• Tercera fase, con aclaramiento central (aspecto en encaje o reticu-

lado). Se resuelve espontáneamente entre la 1.ª y 4.ª semana, pero en ocasiones puede recidivar cuando el niño se estresa, tiene fi ebre, con el sol, etc.

Entre las complicaciones fi guran las artralgias-artritis y la aplasia medular en niños susceptibles, con enfermedades hematológicas, (respuesta 5 correcta).

Pregunta 76.- R: 2Caso clínico de exantema súbito (roséola infantil o sexta enfermedad). Su agente es el herpes virus tipo 6 y afecta principalmente a niños menores de 2 años. Se caracteriza por:• Fase febril: fi ebre alta, con buen estado general, que dura de 3 a 4

días. • Fase exantemática, aparece bruscamente, tras desaparecer la fi ebre,

un exantema maculopapuloso poco confl uente que afecta al tórax, abdomen y raíz de miembros.

Son datos típicos del hemograma la presencia de leucocitosis con neutrofi lia en las primeras 24-36 horas de evolución, por lo que, en un primer momento, puede ser difícil establecer el diagnóstico de infección vírica (respuesta 2 correcta).La complicación más frecuente es la crisis febril. También puede causar encefalitis.

Pregunta 77.- R: 4Recordemos los criterios diagnósticos de la enfermedad de Kawasaki o sd. mucocutáneo-ganglionar.

A Fiebre

B

Presencia de, como mínimo, cuatro de los cinco siguientes signos:· Conjuntivitis bilateral no purulenta· Alteraciones en la mucosa de la orofaringe, con inyección faríngea;

labios secos con fi suras, inyectados o ambos, y lengua “en fresa”· Alteraciones en las zonas periféricas de las extremidades, como

edema y eritema en manos o pies, descamación de inicio periungueal

· Exantema, de inicio en el tronco; polimorfo, no vesicular· Linfadenopatía cervical unilateral

C La enfermedad no se puede explicar por ninguna otra causa conocida

Diagnóstico clínico A + B + C

Pregunta 77. Criterios diagnósticos de la enfermedad de Kawasaki.

Además de estos síntomas, se puede encontrar patología muy variada: artritis, pericarditis, iritis, etc. Es muy llamativa la trombocitosis, espe-cialmente en la fase subaguda de la enfermedad.Recuerda que la etiología es desconocida aunque se plantea que pue-de ser secundaria a un daño inmunitario del endotelio mediado por superantígenos.El tratamiento consiste en el empleo de AAS y gammaglobulina en la fase aguda, manteniendo posteriormente el AAS en dosis antiagregantes. El empleo de la gammaglobulina en la fase aguda previene el desarrollo de aneurismas coronarios en la fase subaguda.

Pregunta 78.- R: 2Este paciente presenta una púrpura de Schölein-Henoch. Es la vasculitis más frecuente de la infancia. Afecta a pequeños vasos y está mediada por mecanismo inmunológico de predominio IgA.

El período de máxima transmisión abarca desde 7 días antes de apari-ción del exantema hasta 7 u 8 días después de que éste haya aparecido.Las complicaciones son poco frecuentes en la infancia: artritis (preferente-mente articulaciones de pequeño tamaño), encefalitis y púrpura trombo-pénica. La rubéola suele darse en niños mayores de 6 meses, pues antes los anticuerpos maternos actúan como protectores (respuesta 3 correcta). Una vez superada la enfermedad suele quedar inmunidad permanente.

Pregunta 73.- R: 1El cuadro clínico descrito es el típico de mononucleosis infecciosa. La clínica característica es astenia importante, fi ebre alta, adenopatías, faringoamigdalitis con exudado blanquecino en sábana, hepatoespleno-megalia y puede cursar con exantema maculopapuloso de predominio en tronco, a veces desencadenado por la toma de amoxicilina. En la analítica típicamente aparece leucocitosis con linfocitosis, presencia de linfocitos atípicos, leve trombopenia y discreto aumento de las transa-minasas, pero puede cursar con pancitopenia como es el caso. El virus Epstein-Barr, los virus A y B de la hepatitis, el parvovirus B19, el VIH y el citomegalovirus, son los más asociados a pancitopenia. Para confi rmar el diagnóstico habría que pedir serología específi ca, aunque el tratamiento es sintomático, y en cuanto a la pancitopenia, actitud expectante, pues se resuelve espontáneamente en la mayoría de las ocasiones.La pancitopenia puede ser congénita o adquirida. La más frecuente es esta última, y las causas más importantes son los fármacos, los tóxicos, las infecciones, radiaciones y procesos inmunitarios. Las leucemias y la hemoglobinuria paroxística nocturna (HPN) son etiologías más raras.

Pregunta 74.- R: 1El caso que se nos presenta es una varicela. Ya sabemos que el agente causal es el virus de la varicela-zóster, de la familia de los herpes virus. La fase prodrómica consiste en un catarro de vías altas con febrícula. Lo característico del exantema es que suele afectar a tronco y parte proximal de extremidades, respetando las zonas distales. Las lesiones aparecen en distintos estadios evolutivos (o lesiones en “cielo estrellado”); éstas son lesiones eritematosas con vesículas y otras en fase de costra. Las lesio-nes pican mucho. La complicación más frecuente es la sobreinfección bacteriana de las lesiones.

Otras complicaciones son:• Neumonía varicelosa: en la infancia es más frecuente la secundaria

a sobreinfección bacteriana, que la producida por el propio virus.• Afectación del SNC: lo más habitual es en forma de cerebelitis, que

evoluciona de forma favorable.

La varicela en los niños sanos no precisa tratamiento con aciclovir. Son indicaciones para su uso: neonatos e inmunodeprimidos.

Pregunta 75.- R: 3La pregunta hace referencia al llamado eritema infeccioso, megaloeritema o quinta enfermedad. En el MIR lo suelen preguntar para confundirlo con el exantema súbito, roséola infantil o sexta enfermedad.El eritema infeccioso tiene como agente causal el parvovirus B19 (res-puesta 1 correcta). Es típico de la edad escolar (5-15 años) y predomina en meses de primavera y verano. El periodo de incubación es 1-2 se-manas (respuesta 2 correcta). El cuadro se caracteriza por un periodo prodrómico que puede cursar con febrícula, sin fi ebre alta (respuesta 3 falsa), seguido de la clínica exantemática afebril que evoluciona en tres etapas (respuesta 4 correcta):• Fase inicial con eritema en ambas mejillas (¨niño abofeteado¨).• Fase intermedia de exantema maculopapuloso en tronco, sin afec-

tación de palmas y plantas.

Page 27: Testcom1v Pd

Comentarios de Test a distancia 1.ª vuelta

Pediatría

CTO Medicina • C/Francisco Silvela, 106 • 28002 - Madrid • Tfno. (0034) 91 782 43 30/33/34 • E-mail: [email protected] • www. ctomedicina. 15

Pregunta 81.- R: 5Respecto al SIDA en pediatría, hay que recordar:• La principal vía de transmisión es vertical, durante el parto. El por-

centaje de infectados es aproximadamente del 1%, si se aplican medidas de actuación oportunas. Recuerda que existe transmisión documentada del virus mediante leche materna.

• El periodo de latencia es menor que en adulto. Se subdivide en SIDA precoz (período de latencia de pocos meses, clínica antes del año de vida, predominando las manifestaciones neurológicas, pronóstico malo) y SIDA tardío (más parecido al del adulto, predominando la clínica infecciosa).

• Las manifestaciones clínicas más frecuentes en pediatría, compa-rándolo con los adultos son: cuadros ORL, parotiditis, neumonía intersticial linfoide y manifestaciones neurológicas (calcifi caciones de los ganglios basales, atrofi a cerebral, etc.). La causa más frecuente de muerte es la neumonía por P. carinii.

• Son infecciones menos frecuentes en los niños: TBC, hepatitis B, linfomas y sarcoma de Kaposi.

Pregunta 82.- R: 1El diagnóstico en el hijo de madre VIH positiva es difícil por la presencia de IgG anti-VIH de la madre que pasan a través de la placenta, de forma que el 100% de los niños son seropositivos al nacer. Los niños no infectados ¨falsamente positivos¨ pueden tardar en negativizar anticuerpos 18 me-ses (respuesta 1 falsa), por ello, el diagnóstico va a depender de la edad:Recuerda que es necesario realizar quimioprofi laxis con AZT durante el embarazo, el momento del parto y luego al RN. El tratamiento se basa en la triple terapia, al igual que en los adultos, y para evaluar la evolución, se emplea la carga viral y el cociente CD4/CD8, recordando que la cifra de linfocitos hay que extrapolarla según la edad del niño.

MENORES DE 18 MESES MAYORES DE 18 MESES

1. PCR-ADN + al menos en 2 determinaciones

2. Criterios SIDA

1. Presencia de Ac frente a VIH (ELISA o Western-Blot)

2. Criterios del apartado anterior

Pregunta 82. Diagnóstico de infección VIH en el niño.

Pregunta 83.- R: 3La triple vírica protege contra el virus del sarampión, rubéola y parotiditis. Es una vacuna de virus vivos atenuados.Indicaciones: a los 15 meses y una dosis de recuerdo a los 4 años, a niños y niñas.Efectos adversos: cuadro de fi ebre y exantema, más atenuados que las infecciones frente a las que protege (son virus vivos).Contraindicaciones:• Inmunodeprimidos: excepto en niños VIH+ (sólo si existe inmuno-

depresión importante). • Embarazo.• La alergia al huevo ha dejado de ser una contraindicación absoluta

(sólo si existe anafi laxia).

Pregunta 84.- R: 4Actualmente existen dos tipos de vacunas: polio oral (Sabin) y polio intramuscular (Salk).La polio Sabin es una vacuna de virus vivos atenuados que produce in-munidad local tipo IgA y vacunación comunitaria por contacto fecal-oral. Como efecto adverso más importante destacan algunos casos aislados de polio en niños sanos, lo que ha motivado en los últimos años que se estén potenciando las vacunas de virus inactivados.

La fase prodrómica suele consistir en proceso ORL. Posteriormente puede aparecer, en orden de frecuencia:• Afectación cutánea: púrpura palpable, en miembros inferiores, que

suele cursar en brotes.• Artritis de rodillas y tobillos.• Dolor abdominal cólico, vómitos y deposiciones con sangre.• Afectación renal: hematuria con o sin proteinuria. Éste es el factor

que marca el pronóstico.• Sólo en menos de un 1% se produce afectación del SNC.

El tratamiento consiste en antiinflamatorios para las molestias articulares y en ciclos cortos de corticoides, en caso de dolor ab-dominal intenso. Si existe afectación renal intensa, se emplean inmunosupresores.

Pregunta 79.- R: 2Para el diagnóstico de mononucleosis infecciosa, aparte de la sospecha clínica, podemos encontrar:• Leucocitosis (10.000-20.000) con más de un 20-40% de linfocitos

atípicos (respuesta 2 falsa).• Aumento de las transaminasas (80% de casos) (respuesta 1 correcta).• Anticuerpos heterófi los: son útiles en mayores de 4 años; por debajo

de esta edad, la sensibilidad es inferiror al 20%. Puede permanecer positivo hasta 9 meses después, por lo que no es útil para el diag-nóstico de infección activa.

• Anticuerpos específi cos contra el VEB (respuesta 3 correcta): en fase aguda: la IgM ACV (frente a la cápside viral) se incrementa rápido y se mantiene 4 semanas. La IgG ACV aparece al fi nal de la fase aguda y se mantiene casi toda la vida.

EL VEB se ha asociado con distintos tumores como el linfoma Burkitt, carcinoma nasofaríngeo, síndrome de Duncan y diversos síndromes linfoproliferativos (respuesta 4 correcta). En la fase aguda de la infec-ción puede producirse rotura esplénica ante un traumatismo, por ello, el tratamiento consiste en reposo; en algunos casos puede ser útil la administración de corticoides (obstrucción de la vía aérea alta, anemia hemolítica autoinmune, convulsiones, etc.).

Pregunta 80.- R: 4Estamos ante un cuadro de tos ferina, infección por Bordetella pertussis. La edad con más riesgo es el periodo de lactante, la madre no le da anticuerpos protectores (porque éstos van disminuyendo a lo largo de la vida) y el niño todavía no tiene completa la vacunación frente al microorganismo (respuesta 1 correcta).El periodo de incubación oscila entre 5-10 días (respuesta 5 correcta). La clínica consta de dos fases:• Fase catarral inicial (respuesta 2 correcta): con febrícula, rinorrea y

tos blanda. Este es el periodo de máxima contagiosidad.• Fase de tos paroxística, consiste en accesos de tos, seguido en mu-

chas ocasiones de vómitos y acompañándose de cianosis facial. En los niños mayores es característico el “gallo” inspiratorio. Esta fase puede durar 2-4 semanas (respuesta 3 correcta).

La tos ferina, pese a ser infección bacteriana, cursa con linfocitosis absoluta (respuesta 4 falsa) y en la radiografía de tórax los datos son inespecífi cos (infi ltrados perihiliares). El diagnóstico se basa en el cul-tivo del moco nasal en medio de Bordet-Gengou pero la detección en sangre de IgG antifactor estimulante de los linfocitos es la técnica más sensible y específi ca.Para el tratamiento se utilizan macrólidos, actualmente, de elección la azitromicina.

Page 28: Testcom1v Pd

Comentarios de Test a distancia 1.ª vuelta

CTO Medicina • C/Francisco Silvela, 106 • 28002 - Madrid • Tfno. (0034) 91 782 43 30/33/34 • E-mail: [email protected] • www. ctomedicina.com 16

PediatríaEstá contraindicada en personas inmunodeprimidas y personas que conviven con éstos.La vacuna tipo Salk (i.m.) está elaborada con virus inactivados, es menos eficaz, pero más segura. Es la que actualmente recomienda la AEP. La pauta de administración es a los 2-4-6-18 meses y a los 4 años.

Pregunta 85.- R: 5Lo básico que tenéis que recordar de la vacuna DTP es lo siguiente:• Componentes:

- Difteria: toxoide.- Tétanos: toxoide.- Pertussis: dos tipos: celular y acelular. Esta última con muchos

menos efectos adversos a nivel neurológico. Recomendada por la AEP. La Pertussis celular estaba contraindicada en mayores de 7 años, porque a partir de esa edad hay más riesgos que benefi cios.

• Indicaciones: en el calendario vacunal se administra a los 2-4-6 y 18 meses y a los 4 años se puede administrar DT o DTPa , luego cada 10 años dT. Las primeras dosis se pueden administrar en un mismo preparado con otras vacunas (polio inactivada, Hib, VHB), para mi-nimizar el número de pinchazos.

En pacientes inmunodeprimidos se puede administrar sin riesgo.

Pregunta 86.- R: 4La vacuna del Hib es una vacuna conjugada, muy segura y efi caz. Protege frente a infecciones invasivas (meningitis y epiglotitis). En el calendario vacunal se incluye a los 2-4-6 y 18 meses. Estaría indicada en niños menores de 5 años. Presenta efectos secundarios leves y de corta duración. Contraindicaciones: en edades inferiores a 2 meses por no ser inmunógena.

Pregunta 87.- R: 4La vacuna frente a la hepatitis B está realizada por ingeniería genética. Producen en las personas que la reciben anticuerpos frente al antígeno de superfi cie del virus. En el calendario vacunal se puede administrar con diversas pautas: (0-1-6), (0-2-6) y (2-4-6). Para prevenir la transmisión de la hepatitis B en el neonato, en el caso de que la madre sea portadora, se debe asociar con gammaglobulina específi ca al nacimiento.Está indicada a cualquier edad si el niño no ha sido vacunado previamente (respuesta 4 falsa). Sus contraindicaciones son muy escasas. Recordar que el embarazo no es una contraindicación absoluta, pero se aconseja evitar todo tipo de vacunas siempre que sea posible.

Pregunta 88.- R: 4Quimioprofi laxis frente a los agentes más frecuentes de meningitis.• Meningococo: a todos los contactos, independiente de la edad,

con rifampicina en dosis de 10 mg/k/dosis cada 12 horas, 4 dosis +/- vacuna en caso del meningococo C.

• Hib: niños menores de 5 años sin vacunar y adultos que conviven con menores de 5 años sin vacunar (éstos pueden ser portadores): rifampicina en dosis de 20 mg/k/dosis cada 24 horas, 4 dosis +/-va-cuna.

• Neumococo: no existe quimioprofilaxis activa. Se puede indicar la vacuna del neumococo de 7 serotipos para prevenir futuros contactos.

Pregunta 89.- R: 2En el esquema siguiente aparece esquematizada la actitud que hay que llevar a cabo con un niño que ha tenido un contacto con un bacilífero positivo.

Pregunta 89. Algoritmo de actuación frente a contacto con un bacilífero positivo.

Pregunta 90.- R: 3El SMSL, muerte repentina de un niño menor de un año cuya etiología se desconoce, supone la causa principal de muerte entre el mes de vida y el año de edad en los países desarrollados. Este síndrome se correlaciona con la posición de decúbito prono y lateral para dormir. Tras este factor posicional, el tabaquismo materno es actualmente el principal factor de riesgo del SMSL. Otros factores son la prematuridad, el antecedente de hermano muerto por SMSL, sexo masculino, etc.Debido a la ausencia de tratamiento, es muy importante la prevención con medidas como la posición de decúbito supino para dormir y la lac-tancia materna (que se considera el factor preventivo más importante).

Pregunta 91.- R: 5El ductus arterioso persistente es una cardiopatía típica de RNPT, que consiste en la unión entre la aorta distal a la subclavia izquierda y la arteria pulmonar. Esta cardiopatía congénita también se asocia a la rubéola congénita y es más frecuente en niñas que en niños.La clínica consiste en un soplo continuo, en maquinaria o de Gibson, más audible en 2.º espacio intercostal izquierdo; los pulsos femorales son saltones. El niño puede presentar clínica de ICC.En la Rx de tórax se aprecia plétora pulmonar con aumento de cavidades izquierdas. El diagnóstico defi nitivo lo obtenemos por ecocardiografía. En los RNPT, si no existen contraindicaciones, se comienza con la admi-nistración de indometacina, y si no se soluciona, tratamiento quirúrgico. Recuerda las contraindicaciones de la indometacina: hemorragia activa, trombopenia, enterocolitis necrotizante e insufi ciencia renal. En caso de no ser un RNPT, el tratamiento de entrada es quirúrgico. Cuando interesa mantenerlo abierto se utiliza PGE1 i.v. (respuesta 5 falsa).

Pregunta 92.- R: 4La TGA es la cardiopatía cianógena más frecuente de inicio en el perío-do neonatal (recordad que la más común en conjunto es la tetralogía de Fallot, cuya clínica es más tardía, en torno al año de vida). La aorta se origina en el ventrículo derecho, a la derecha y delante de la arteria pulmonar, que se forma en el ventrículo izquierdo. La clínica suele consistir en un RN que en las primeras horas de vida pre-senta taquipnea y cianosis intensa (conforme se va cerrando el ductus), no tiene soplo y los pulsos son débiles. En la Rx de tórax se aprecia un pedículo cardíaco estrecho con plétora pulmonar. El ECG suele ser de

Page 29: Testcom1v Pd

Comentarios de Test a distancia 1.ª vuelta

Pediatría

CTO Medicina • C/Francisco Silvela, 106 • 28002 - Madrid • Tfno. (0034) 91 782 43 30/33/34 • E-mail: [email protected] • www. ctomedicina. 17

características normales para un RN. El diagnóstico se confi rma mediante la ecografía.El tratamiento consiste en la perfusión de PGE de forma urgente para mantener abierto el ductus, asegurando comunicación entre ambas circulaciones en paralelo para que sea posible la supervivencia y, cuanto antes, realizar la corrección quirúrgica. La técnica de elección es el switch arterial o Jatene, que consiste en seccionar la salida de los grandes vasos e intercambiarlos.

Pregunta 93.- R: 2Los malos tratos en la infancia son la segunda causa de muerte en España durante los primeros 5 años de vida, excluyendo el periodo neonatal. Es muy importante detectarlos precozmente pues hay casos fatales que producen daños importantes al niño, incluso la muerte. La actitud de los padres (despreocupación por la gravedad de las lesiones, justificaciones poco lógicas para las lesiones detectadas, preocupación por la propia situación personal y no por la del niño, etc.), la actitud del niño (ansiedad excesiva al hablarle y explorarle, pasividad inadecuada para su edad, conducta agresiva, distancia o proximidad excesivas con los padres) y la exploración física, son las herramientas que tenemos para sospechar el diagnóstico. De la exploración, son indicadores muy sugerentes: ETS en niñas prepúbe-res, embarazo en adolescente joven, lesiones genitales o anales sin traumatismo previo; son indicadores inespecíficos: dolor o sangrado vaginal o rectal, enuresis, encopresis, dolor abdominal, lesiones traumáticas en distintas fases de evolución, lesiones en localizacio-nes que no corresponden con la actividad del niño ni su desarrollo psicomotor, fracturas raras en niños como la del acromión. Ante la sospecha fundada, es obligatorio dar parte a la Autoridad Judicial y ponerse en contacto con los Servicios Sociales y de Protección y Atención al Menor, y ampliar el estudio con una serie ósea completa, un examen del fondo de ojo, una TC craneal, entre otras.

Pregunta 94.- R: 3La CoA consiste en un estrechamiento de la luz aórtica. Recuerda que es más frecuente en niñas con síndrome de Turner. El tipo más común es yuxtaductal (respuesta 1 correcta).Los niños suelen estar asintomáticos (respuesta 2 correcta) y la enferme-dad puede pasar desapercibida hasta la edad adulta. En la exploración es típico encontrar pulsos en extremidades inferiores disminuidos y retrasados en comparación con los miembros superiores, lo que supo-ne HTA en miembros superiores (respuesta 3 falsa) e incluso puede ser mayor la TA en el miembro derecho que en el izquierdo, sobre todo si la coartación es proximal a la salida de la subclavia izquierda (respuesta 4 correcta). En la Rx de tórax son típicas las escotaduras costales en los bordes inferiores de las costillas o signo de Rösler (respuesta 5 correcta) y, a veces, el llamado signo del 3 o escotadura de la aorta en el lugar de la coartación. En el ECG aparecen signos de hipertrofi a de VI. El tratamiento en niños es quirúrgico.

Pregunta 95.- R: 2Pregunta sobre el desarrollo puberal normal. Recuerda que la pubertad en niñas se inicia a partir de los 9 años y en niños a partir de los 11.El aumento del tamaño testicular en el varón es el primer signo de pu-bertad (respuesta 2 correcta). En las niñas, el primer dato a la exploración sugestivo de inicio de la pubertad, es la aparición de botón mamario.La ganancia de talla es más acusada en niños que en niñas.

Pregunta 96.- R: 3La telarquia prematura es el desarrollo mamario prematuro en niñas sin otros caracteres sexuales secundarios, sin aceleración de la velocidad

de crecimiento ni de la edad ósea. La etiopatogenia no está clara, y es probable que sea multifactorial. Es más frecuente en los dos primeros años de vida, y en general tiene buen pronóstico, con regresión espon-tánea en la mayoría de los casos en 2-3 años. Sin embargo, a veces es persistente, pudiendo evolucionar a una pubertad precoz o adelantada, sin que se conozcan actualmente los factores de riesgo para ello. Para el diagnóstico son útiles:• Rx de mano y muñeca izquierdas (edad ósea).• Ecografía pélvica.• Niveles basales de estradiol.• La prueba diagnóstica para excluir una pubertad precoz es el test

de GnRH (respuesta de FSH y LH ante el estímulo con análogos de GnRH).

Pregunta 97.- R: 4Existen numerosos síndromes genéticos y polimalformativos que se ma-nifi estan con talla baja. Sin embargo, el síndrome de Klinefelter (47xxy) se caracteriza por presentar talla alta junto con otras alteraciones (retra-so mental, alteraciones de la conducta, DM, alteraciones de la función tiroidea, testes pequeños e infertilidad, etc.).

Recuerda las principales características de los otros síndromes:• Síndrome de Turner (45x0): talla baja, Pterigium colli, ausencia de

desarrollo puberal, etc. Puede asociar cardiopatía y alteraciones renales.

• Síndrome de Down (95% casos, trisomía 21): hipotonía, hendiduras palpebrales, aumento de piel en zona nucal, retraso mental variable. Puede asociar cardiopatía (siendo típico el canal AV).

• Síndrome de Silver-Russell: cuadro de talla baja que se manifi esta desde la etapa fetal y aspecto pseudohidrocefálico.

• Síndrome de Noonan: talla baja, epicantus, hipertelorismo ocular, raíz nasal hundida, hipertelorismo mamilar. Es característica la estenosis pulmonar.

Pregunta 98.- R: 1La causa más frecuente de convulsiones en el RN es la encefalopatía hipoxicoisquémica. Las crisis suelen ser focales, no generalizadas, puesto que el SNC del neonato no está completamente mielinizado. El diagnóstico se basa en el EEG. El fenobarbital es el fármaco de pri-mera elección. El pronóstico dependerá de la causa desencadenante (respuesta 1 falsa).

Pregunta 99.- R: 3La enuresis es la emisión involuntaria de orina en la ropa o en la cama después de la edad en que los niños deben tener control de esfínteres (5 años de edad), en ausencia de patología orgánica. Su prevalencia es mayor en niños que en niñas (respuesta 1 falsa). Existe un claro patrón familiar especialmente importante para la enuresis nocturna (respuesta 3 correcta).El 90% de los casos son primarios o persistentes (el niño no ha llegado nunca a tener un control completo de la orina) y se deben a un trastorno madurativo. Los casos secundarios son más frecuentes entre los 5 y los 8 años de edad, y característicamente son transitorios y de buen pronóstico (respuesta 2 falsa). La enuresis diurna es más frecuente en niñas, y casi siempre se debe al aplazamiento de la micción (hábito retentor) hasta rebasar la capacidad de la vejiga (respuesta 4 falsa). El tratamiento más correcto debe buscarse después de una evaluación psicosocial y la exploración física. Lo más efectivo son las técnicas conductuales. La desmopresina es efectiva pero, asimismo, tiene una alta tasa de recaídas, y supone un riesgo de retención hídrica e hipo-natremia, por lo que se recomienda sólo en situaciones puntuales.

Page 30: Testcom1v Pd

Comentarios de Test a distancia 1.ª vuelta

CTO Medicina • C/Francisco Silvela, 106 • 28002 - Madrid • Tfno. (0034) 91 782 43 30/33/34 • E-mail: [email protected] • www. ctomedicina.com 18

PediatríaPregunta 100.- R: 2El cuadro clínico hace referencia a un niño con una convulsión febril. La clínica típica consiste en fi ebre alta, crisis tónico-clónica generalizada de breve duración con un periodo postcrítico corto. La edad de pre-

sentación es entre los 6 meses y 5 años. Existen antecedentes familiares hasta en un 30% de los casos. El tratamiento en la fase aguda consiste en la administración de diazepam intrarrectal y medidas para disminuir la hipertermia.

Page 31: Testcom1v Pd